Download as pdf or txt
Download as pdf or txt
You are on page 1of 212

Table of Contents

Chapter 1: Place Value, Multiplication and Expressions .............................. 7


Lesson 1: Place Value and Patterns ................................................................... 8
Lesson 2: Place Value of Whole Numbers .................................................... 10
Lesson 3: Properties .............................................................................................. 13
Lesson 4: Powers of 10 and Exponents ......................................................... 15
Lesson 5: Multiplication Patterns .................................................................... 17
Lesson 6: Multiply by 1-Digit Numbers ..................................................... 19
Lesson 7: Multiply by 2-Digit Numbers ..................................................... 22
Lesson 8: Relate Multiplication to Division ................................................. 24
Lesson 9: Multiplication and Division ............................................................ 26
Lesson 10: Numerical Expressions.................................................................. 28
Lesson 11: Evaluate Numerical Expressions .............................................. 32
Lesson 12: Grouping Symbols ........................................................................... 34
Chapter 2: Divide Whole Numbers...................................................................... 37
Lesson 1: Place the First Digit............................................................................ 38
Lesson 2: Divide by 1-Digit Divisors ............................................................... 40
Lesson 3: Division with 2-Digit Divisors ................................................... 42
Lesson 4: Partial Quotients ................................................................................. 44
Lesson 5: Estimate with2-Digit Divisors ....................................................... 46
Lesson 6: Divide by 2-Digit Divisors ............................................................... 48
Lesson 7: Interpret the Remainder ................................................................. 51
Lesson 8: Adjust Quotients ................................................................................. 53
Lesson 9: Problem Solving: Division............................................................... 55
Chapter 3: Add and Subtract Decimals .............................................................. 57
Lesson 1: Thousandths ......................................................................................... 58
3
Lesson 2: Place Value of Decimals................................................................... 60
Lesson 3: Compare and Order Decimals ...................................................... 64
Lesson 4: Round Decimals.................................................................................. 67
Lesson 5: Decimal Addition ............................................................................... 69
Lesson 6: Decimal Subtraction ......................................................................... 71
Lesson 7: Estimate Decimal Sums and Differences ................................. 73
Lesson 8: Add Decimals ....................................................................................... 75
Lesson 9: Subtract Decimals .............................................................................. 77
Lesson 10: Patterns with Decimals................................................................. 80
Lesson 11: Problem Solving: Add and Subtract Money ......................... 82
Lesson 12: Choose a Method ............................................................................. 85
Chapter 4: Multiply Decimals................................................................................ 87
Lesson 1: Multiplication Patterns with Decimals ..................................... 88
Lesson 2: Multiply Decimals and Whole Numbers .................................. 90
Lesson 3: Multiplication with Decimals and Whole Numbers ............ 92
Lesson 4: Multiply Using Expanded Form ................................................... 95
Lesson 5: Problem Solving: Multiply Money .............................................. 97
Lesson 6: Decimal Multiplication .................................................................... 99
Lesson 7: Multiply Decimals ............................................................................ 101
Lesson 8: Zeros in the Product ....................................................................... 103
Chapter 5: Divide Decimals.................................................................................. 105
Lesson 1: Division Patterns with Decimals ............................................... 106
Lesson 2: Divide Decimals by Whole Numbers ....................................... 108
Lesson 3: Estimate Quotients.......................................................................... 110
Lesson 4: Division of Decimals by Whole Numbers .............................. 112
Lesson 5: Decimal Division .............................................................................. 114
Lesson 6: Divide Decimals ................................................................................ 116
Lesson 7: Write Zeros in the Dividend ........................................................ 118
Lesson 8: Problem Solving: Decimal Operations .................................... 120

4
Chapter 6: Add and Subtract Fractions with Unlike Denominators .. 123
Lesson 1: Addition with Unlike Denominators ....................................... 124
Lesson 2: Subtraction with Unlike Denominators ................................. 126
Lesson 3: Estimate Fraction Sums and Differences .............................. 128
Lesson 4: Common Denominators and Equivalent Fractions ........... 131
Lesson 5: Add and Subtract Fractions......................................................... 134
Lesson 6: Add and Subtract Mixed Numbers ........................................... 137
Lesson 7: Subtraction with Renaming ........................................................ 141
Lesson 8: Patterns with Fractions ................................................................ 143
Lesson 9: Practice Addition and Subtraction and Properties of
Addition.................................................................................................................... 145
Chapter 7: Multiply Fractions ............................................................................. 149
Lesson 1: Fraction Multiplication ................................................................ 150
Lesson 2: Multiply Mixed Numbers.............................................................. 157
Chapter 8: Divide Fractions ................................................................................. 161
Lesson 1: Interpret Division With Fractions ............................................ 162
Chapter 9: Algebra: Patterns and Graphing ................................................ 165
Lesson 1: Line Plots............................................................................................. 166
Lesson 2: Graph Data.......................................................................................... 169
Lesson 3: Line Graphs ........................................................................................ 172
Lesson 4: Graph and Analyze Relationships............................................. 174
Chapter 10: Convert Units of Measure............................................................ 179
Lesson 1: Customary Conversions in Multi-step Problems ............... 180
Lesson 2: Problem Solving: Customary and Metric Conversions .... 186
Lesson 3: Elapsed Time ..................................................................................... 191
Chapter 11: Geometry and Volume .................................................................. 193
Lesson 1: Polygons .............................................................................................. 194
Lesson 2: Triangles.............................................................................................. 196
Lesson 3: Quadrilaterals ................................................................................... 199

5
Lesson 4: Solid Figures and Volume............................................................. 202
Lesson 5: Comparing Volumes and Composite Figures....................... 208

6
C
HAPTER 1
Chapter 1: Place Value,
Multiplication and Expressions

7
Chapter 1 Lesson 1: Place Value and
Patterns
GO MATH: Ch.1 L1

Vocabulary Keywords Learning Outcomes:

Place Value -Recognize the place value of ones,


Pattern tens, hundreds and thousands.
Describe the relationship between
two place value positions.

Consider the table below.

Number 1,000 100 10 1

Model

Do you notice any pattern?

Rule: Any place value position is always 10 times greater than the
position to its right and any place value position is 1/10 of the position to
its left.

Example:Complete the table below.


Number 10 times as 1/10 of
much as
20
90
8000

Step 1: Write the number in a place value chart.


Step 2: Write a number that is 10 times as much as the number given
using the place value chart.
Step 3: Write the number that is 1/10 of the given number using the
place value chart.

8
Number 10 times as 1/10 of
much as
20 200 2
90 900 9
8000 80,000 800

Exercises 1 – 7:

A- Complete the table below using the place value chart.


1)
Number 10 times as 1/10 of

much as
300
40
12000

2)
Number 10 times as 1/10 of
much as
700
70,000
5,000

B- Complete the sentence with 100 or 1000.

3) 300 is ___ times as much as 3.


4) 600,000 is _____ times as much as 600.
5) 70,000 is _____ times as much as 70

C- Complete the sentence.

6) 90,000 is 1/10 of _____


7) 800 is 10 times as much as _____

9
Chapter 1 Lesson 2: Place Value of Whole
Numbers
GO MATH: Ch.1 L2

Vocabulary Keywords: Learning Outcomes:

Place value -Determine the place value of a


Standard form digit
Word form
Expanded form -Write a number in standard form,
Period word form or expanded form

We can write numbers in three different forms: standard, expanded


and word form.

You can use a place value chart to help you read and write greater
numbers such as 271,603,894.

MILLIONS THOUSANDS ONES


H T O H T O H T O
2 0 0 , 0 0 0 , 0 0 0
7 0 , 0 0 0 , 0 0 0
1 , 0 0 0 , 0 0 0
6 0 0 , 0 0 0
0 0 , 0 0 0
3 , 0 0 0
8 0 0
9 0
4

Standard form:271,603,894

Expanded form:200,000,000 + 70,000,000 + 1,000,000 + 600,000 +


3,000 + 800 + 90 + 4

Word form: Two hundred seventy-one million, six hundred three


thousand, eight hundred ninety-four.

10
Example 1:Write the value of the underlined digit.

1) 288,726,147 2) 634,175,843
200,000,000 4,000,000

3) 97,463 4)25,908,167
90,000 20,000,000

Example 2:Write the number in two other forms.

1) 647,200
Expanded form: 600,000 + 40,000 + 7,000 + 200
Word form: six hundred forty seven thousand, two hundred

2) 40,000,000 + 20,000 + 1,000 + 80 + 5


Standard form: 40,021,085
Word form: forty million, twenty one thousand, eighty five

3)Fifty billion, three hundred sixty four million, three hundred two
Standard form: 50,364,000,302
Expanded form: 50,000,000,000 + 300,000,000 + 60,000,000 +
4,000,000 + 300 + 2

Example 3:What number makes the statement true?

1) 580,000 = 58 x 10,000 2) 2,760,000 = 276 x 10,000

3) 6,420,000 = 642 x 10,000 4) 57,910,000 = 5,791 x 10,000

Example 4:The population of Dubai in


2005 is 1,204,000. How can Rima write
the population of Dubai in expanded
form?

The population of Dubai in expanded form


is:
1,000,000 + 200,000 + 4,000

11
Exercises1 - 10:

A- Write the value of the underline digit

1) 42,725,189 2) 3,927,532 3) 12,725,693

4) 7,981,325 5) 59,798,612 6) 65,127,382

B- Write each number in two different ways.

7) 4,000,000 + 60,000 + 5,000 + 40 + 3

8) Two million, sixty- five thousand, two hundred fifty seven

9) 11,470,351

10) 79,865,321

12
Chapter 1 Lesson 3: Properties

GO MATH: Ch.1 L3

Vocabulary Keywords: Learning Outcomes:


Commutative Property
Associative Property -Recognize the properties of
Identity Property operations.
Distributive Property -Use the properties of operations to
solve problems

Rules and regulations are very important because


these serve as a guide to what is acceptable or not in a
society. Without rules, there will be chaos in the
community because nothing controls us or guide us on
what is right and what is wrong. Similarly,
mathematics is guided with several rules. Most of them
are stated as properties.

Properties of operations are used to evaluate numerical expressions


more conveniently.
Properties of Addition
Commutative Property: Changing the order 10 + 6 = 6 + 10
of the addends will not affect the sum.
Associative Property: Grouping the addends 2 + (4 + 5) = (2 + 4) + 5
will not affect the sum.
Identity Property: Any number added to 0 is 14 + 0 = 14
equal to the same number.

Properties of Multiplication
Commutative Property: Changing the order 10 × 6 = 6 × 10
of the factors will not affect the product.
Associative Property: Grouping the factors 2 × (4 × 5) = (2 × 4) × 5
will not affect the product.
Identity Property: Any number multiplied to 14 × 1 = 14
1 is equal to the same number.

13
Distributive Property
The distributive property lets you 3 (4 + 5)= (3 × 4) + (3 × 5)
multiply a sum by multiplying each
addend separately and then add the 2 (15 - 7)= (2 × 15) - (2 × 7)
products.

Example 1:Use the properties to find the 39 + 48 + 11.

39 + 48 + 11 = 48 + 39 + 11 Commutative Property

= 48 + (39 + 11) Associative Property

= 48 + 50 Add mentally

= 98 Add mentally

Example 2:Use Distributive Property to find 7 × 98.

7 × 98 = 7 × (100 – 2) Rewrite 98 as 100 – 2


using a multiple of 10.
= (7 × 100) – (7 × 2) Distributive Property

= 700 - 14 Multiply mentally

= 686 Subtract mentally

Exercises 1 – 14:

A- Determine which property of addition or multiplication is


shown (Associative, Identity, Distributive or Commutative).
1) ) 4 +0 = 4 2) (5 × 7) × 4 = 5 × (7 × 4)
3) 10 + 8 = 8 + 10 4) 4 × (5 + 6) = (4 × 5) + (4 × 6)
5) 6 × (5 + 8) = (6 × 5) + (6 × 8) 6) (8 × 3) × 6 = 8 × (3 × 6)
7) 7 + (2 + 3) = (7 + 2) + 3 8) 8 × 1 = 8
9) (1× 10) + (1× 3) = 1 × (10+3) 10) 1 × (2 × 4) = (1 × 2) × 4

B- Use the properties to find the sum or product.

11) 4 × 999 12) 12 × 99, 998


13) 18 + 36 + 82 14) 23 + 72 + 77 + 28

14
Chapter 1 Lesson 4: Powers of 10 and
Exponents
GO MATH: Ch.1 L4

Vocabulary Keywords: Learning Outcomes:

Base -Show powers of 10 using


Exponent exponents.
-Identify the base and the exponent.

Consider the following equations.


10 x 1 = 10
10 x 10 = 100
10 x 10 x 10 = 1,000
10 x 10 x 10 x 10 = 10,000
Do you notice any pattern in these equations? Notice that the number of
zeros in the product is equal to the number of times that 10 is multiplied
by itself. In the last equation, 10 is the repeated factor and the number of
times this number is multiplied by itself is 3. One way to relate these two
is with the use of exponents.

Definition 1: The number that is used as a repeated


factor is called the base.

Definition 2: The number that tells the number of


times the base is used as a factor is called the
exponent.

Different Ways to Write Powers of Ten:

Exponent form Expanded form Standard form Word form

108 108 = 10 × 10 × 10 100, 000, 000 One hundred


× 10 × 10 × 10 million
× 10 × 10

15
Example 1:Write in exponent form and word form.

1) 10 x 10 x 10
Exponent Form: 103
Word Form: the third power of ten

2) 10 x 10 x 10 x 10
Exponent Form: 104
Word Form: the fourth power of ten

Example 2:Find the value.

1) 2 x 103 2) 6 x 108
= 2 x 1,000 = 6 x 100,000,000
= 2,000 = 600,000,000

Exercises 1 – 6:

A- Write in exponent form and word form.


1) 10 x 10 x 10 x 10 x 10 2) 10 x 10 x 10 x 10 x 10 x 10

B- Find the value.


3) 107 4) 3 x 104

C- Complete the pattern.


5) 8 x 100 = 8 x 1 = _______
8 x 101 = 8 x 10 = _______
8 x 102 = 8 x 100 = _______
8 x 103 = 8 x 1,000 = _______
8 x 104 = 8 x 10,000 = _______
D- Solve.
6) The diameter of the planet Saturn plus its ring is about 300,000
kilometers. Express this number as a whole number multiplied by
the power of ten.

16
Chapter 1 Lesson 5: Multiplication Patterns

GO MATH: Ch.1 L5

Vocabulary Keywords: Learning Outcomes:


Multiply -Use a basic fact to multiply by a 2
Pattern –digit number.
- Use a pattern to multiply by a 2 –
digit number.

Consider the pattern below.


11 x 11 = 121
12 x 11 = 132
13 x 11 = 143
14 x 11 = 154
We can use this pattern to find the product of 15 x 11. Continuing the
pattern, we will get a product of 165.
In this lesson, we will deal with patterns that are useful in multiplying 2
digit numbers.

Rule: To use the basic fact and a pattern to multiply by a 2-digit number,
start with a basic fact and then write more equations with additional
zeros in factors until you obtain the product of the original problem.
Check for patterns to write the product.

Example 1:Use mental math and a pattern to determine the product


of 12 and 2000.
To find the product of these two numbers, we can use the basic fact
12 x 2 = 24 then add three zeros. Therefore, 12 x 2,000 = 24,000.

Example 2:Use mental math to complete the pattern.


9 x 6 = 54
(9 x 6) x 101 = ______
(9 x 6) x 102 = ______
(9 x 6) x 103 = ______
(9 x 6) x 104 = ______

17
Multiply 9 x 6 and then write the corresponding number of zeros in each
product.
So,
9 x 6 = 54
(9 x 6) x 101 =540
(9 x 6) x 102 =5,400
(9 x 6) x 103 = 54,000
(9 x 6) x 104 =540,000

Exercises 1- 8:

A- Use mental math and a pattern to determine the product.


1) (3 x 7) x 102 2) (4 x 8) x 103
3) (8 x 6) x 105 4) (8 x 9) x 106

B- Use mental math to complete the pattern.


5) 8 x 5 = 40 6) 3 x 9 = 27
(8 x 5) x 101 = ______ (3 x 9) x 101 = ______
(8 x 5) x 102 = ______ (3 x 9) x 102 = ______
(8 x 5) x 103 = ______ (3 x 9) 103 = ______
(8 x 5) x 104 = ______ (3 x 9) x 104 = ______

C- Solve.
7) A Quadrillion has a common notation of 1,000,000,000,000,000. If
I have 4 quadrillion, how much is that? Express your answer as
the product of a number and a power of 10.
8) The total number of human beings on Earth was estimated to be
more than 7 x 109. What is the Earth’s human population in
standard form?

18
Chapter 1 Lesson 6: Multiply by 1-Digit
Numbers
GO MATH: Ch.1 L6

Vocabulary Keywords: Learning Outcomes:


-Compute the product of a number and
Multiply a 1 – digit number
Place Value -Solve problems involving
Product multiplication of a number by a 1 –
Factor digit number.

The teacher has 58 pages hand-outs for his 9 students who are taking
remedial classes. How many papers will he need if he wants to
distribute copies of the hand-outs to his 9 students?
To solve this problem, we have to get the product of the number of
pages in one copy by the number of students. But how can we multiply
such numbers?

Rule: To multiply multi-digit numbers by a 1-digit number, use place


value and regrouping, Start by multiplying the ones, then the tens, and
so on. Use regrouping as necessary.

Example 1:Multiply 143 by 3.


Multiply by ones. Multiply by tens. Multiply byhundreds.

143 143 143


× 3 × 3 × 3
9 29 429
Sometimes you need to regroup.

Step 1:Multiply the ones. 3 × 4 tens= 12 tens 143


× 3_
9
1
Step 2:Multiply the tens. 3 × 4 tens= 12 tens 143
× 3_
29
Write the 2. Regroup the 10 tens as 1 hundreds.
19
Step 3:Multiply the hundreds. 3 × 1 hundreds 143
= 3 hundreds × 3_
429
Now add the regrouped hundred.
3 hundreds + 1 hundred = 4 hundreds
So, 3 × 143 = 429.

Example 2:Find the product.


1) 54 2) 607 3) 3, 719
  x  3   x  8   x  7
162 4,856 26,033

4) 28, 421
  x  5
142,105

Example 3:Solve.
DNATA Offers: Use the table below to answer the questions

Destination Cost in Aed


Costa Del Sol 5,699
Rome 4,899
Paris 5,299
Athens 4,499

1) How much would it cost a family of 4 to fly to Paris?


5,299 x 4 = 21,196 Aed

2) How much would it cost a family of 5 to fly to Athens?


4,499 x 5 = 22,495 Aed

3) How much would it cost a couple to fly to Costa Del Sol?


5,699 x 2 = 11,398 Aed

20
Exercises 1-6: Find the product.

1) 725 2) 2,251 3) 35,798


  x  7   x  4   x  5

4) 692 5) 32,525
  x  9   x  6

6) Challenge yourself: Multiplications cross.

21
Chapter 1 Lesson 7: Multiply by 2-Digit
Numbers
GO MATH: Ch.1 L7

Vocabulary Keywords: Learning Outcomes:


-Calculate the product of a multi-digit
Multiply numbers and 2 – digit numbers.
Place Value -Identify the unknown digit in a
Product multiplication problem.
Factor

In the previous lesson, we have learned how to multiply a multi-digit


number by a 1 – digit number. This time, we will multiply whole
numbers by 2-digit numbers.

Rule: You can multiply by two-digit numbers by breaking apart one of


the factors.

Example 1:Find 21 × 14

You can break the 14 into 1 ten 4 ones.


10 + 4
Step 1:Multiply by the ones.
21
× 4_
84

Step 2:Multiply by the tens. 21


21
× 10_
210

Step 3:Add the products.


21
× 14_
84 ←4 × 21
+ 210_ ←10 × 21
294
So, 21 × 14 = 294.

22
Example 2:Find the product.

1) 541 2) 7, 936 3)
x  64 x  88 1,528
2,164 63,488 x  93
+ 32,460 + 634,880 4,584
34,624 698,368 + 137,520
142,104

Example 3:Last year, Khalid cycled 19 miles every day for a full year,
or 365 days. How many miles did he cycle last year?
365 x 19 = 6,935 miles

365
x  19
3,285
+ 3,650
6,935

Exercises 1-4:
A- Find the product.
1) 726 2) 635 3) 7,392
  x  54   x  85   x  72

B- Fill in the missing number.


4)
327
× 8

26
98

23
Chapter 1 Lesson 8: Relate Multiplication to
Division
GO MATH: Ch.1 L8

Vocabulary Keywords: Learning Outcomes:


Inverse Operation -Use multiplication to solve
Multiplication division problems.
Division -Recognize that multiplication and
division are inverse operations.

We are already familiar of the fact that 4 + 5 = 9 and 9 – 5 = 4 are true


statements involving addition and subtraction. In this lesson, we will
relate multiplication and division and solve problems involving them.

Definition: Opposite operations that undo each other are called inverse
operation. Addition and subtraction are inverse operations.
Multiplication and division are inverse operations.

Rule: To use multiplication in solving division problems, the first step is


to draw a model and divide it into two portions and write the divisor on
the left side. Then write two multiples of the divisor whose total is equal
to the dividend. Find the quotient of those numbers and write the
quotients. The sum of the quotients is the final answer.

Example: Find the quotient of 48 and 6.

Step 1: Draw an area model. Write a related multiplication sentence for


the problem. Use the divisor as a factor and let the dividend be the
product. The quotient will be the unknown value.
?
48 ÷ 6 = ____
6 x _____ = 48 6 48

6x ? = 48

24
Step 2: Use the Distributive Property in order to break apart the large
area into smaller areas for partial products you are familiar with.

? ?

(40 + 8) = 48 4 40 8

(4 x 10) + (4 x 2) = 48 (4 x ?) + (4 x ?) = 48

Step 3: Add the unknown factors of the smaller areas.

10 + 2 = 12

Step 4: Write the multiplication sentence with the unknown factor that
you got. Then use the multiplication sentence to get the quotient.

4 x 12 = 48

48 ÷4 = 12

Exercises 1 – 8:

A- Use multiplication and Distributive Property to find the


quotient.

1) 32 ÷ 4 2) 56 ÷ 8

3) 72 ÷ 3 4) 64 ÷ 4

5) 63 ÷ 3 6) 57 ÷ 3

B- Solve.

7) Omar was at the beach for five days and found 24 seashells. He
plans to give all of his seashells equally to his four siblings. How
many seashells will each friend get?

8) There were a total of 18 football games during the three month


season. If the games are equally divided, how many football games
are played a month?

25
Chapter 1 Lesson 9: Multiplication and
Division
GO MATH: Ch.1 L9

Vocabulary Keywords: Learning Outcomes:


-Solve a simpler problem to solve
Multiplication division problems.
Division - Solve a simpler problem to solve
multiplication problems.

We have learned last time on how to use the Distributive Property to


solve division problems related to multiplication. This time we will deal
with more problems involving them.

Rule: Use the strategy “solve a simpler problem” to break apart a


dividend into smaller quantities that are simpler to divide in order to
solve division problems.

Example: At a pet shop, Mohammad has to feed his 6 Siberian


Huskies for breakfast. He only has available eight 24-ounce cans of
dog food. How is he going to divide the available dog food equally
with the dogs?

Step 1: Read the problem carefully.

Step 2: Identify what you need to find.

In this problem, we have to determine the number of ounces of


dog food that each Siberian husky gets.

Step 3: Determine the information that you need to use.

We need to use the number of cans of dog food, the number of


ounces in each can and the number of dogs.

Step 4: Determine how you will use the information.

We can multiply to determine the total number of ounces. Then


use the strategy “solve a simpler problem” to divide the total by 6

26
Step 5: Solve the problem.

Multiply to find the total number of ounces of dog food available.

8 x 24 = 192

Divide to find the number of ounces each dog gets.

192 ÷ 6 = _____

Break 192 into 2 simpler numbers that are easier to work on to find the
quotient.

192 ÷ 6 = _____

(120 + 72) ÷ 6 = _____

(120 ÷ 6) + (72 ÷ 6) = ______

20 + 12 = 32

Therefore, each Siberian husky will get 32 ounces of dog food.

Exercises 1 – 3:

1) A football team has $448 to spend on caps for their fans. Each cap
costs $8. How many caps can the team buy?

2) There are 185 chairs set up in the gym for a meeting. After the
meeting, the monobloc chairs will be put away in stacks of 5. How
many stacks will there be?

3) A basketball team wants to order 231 T-shirts for the fans. If there
are 3 T-shirts in each package, how many packages of T-shirts
should the team order?

27
Chapter 1 Lesson 10: Numerical
Expressions
GO MATH: Ch.1 L10

Vocabulary Keywords: Learning Outcomes:


- Translate an English expression
Numerical Expression into a mathematical expression.
Operation -Recognize patterns and describe
patterns in words.

If someone asks you, “how much is your change in the cashier if I paid
100AED and the cost of the item I bought is 65 AED?” How are you going
to solve the problem? Sometimes, it would be better to translate words
into expressions in order for us to solve a certain problem easily.

In this lesson we will translate mathematical expressions into numerical


expressions.

Definition: A numerical expression has numbers and operation signs.


It does not have an equal sign.

Key words and phrases and their translations:

 Exceeds
 Increased by
 More than
 Combined, together
Total of
Addition + 
 Sum
 Added to
 In addition
 In excess
 Greater

28
 Diminished by
 Decreased by
 Minus
 Less
Subtraction
- 


Difference between/of
Less than
Fewer than
 Subtract
 Reduced
 Remainder

 Of
Multiplication  

times
multiplied by
 product of

 Per
 A
 out of
Division  

ratio of
quotient of
 Divide
 into

Example 1:Translate the following into an expression.


1) Jaffar had 14 marbles. He won 6 more.
Clue word: more
14 + 6

2) Maya had $20. She spent $5.


Clue word: spent
20 – 5

3) Wassim won 7 medals in 2014. He won 5 more


in 2015.

more is the clue: 7 + 5

29
4) Sophie found out that she has 24 dresses in her wardrobe. She gave
11 of her dresses to her cousin.

gave is the clue: 24 – 11

Example 2:Which problem matches the numerical expression


24 – (6 + 7)?

Problem 1: Abdulla has 24 AED to spend for his evening snack. He


spends 6 AED for the drinks. Then his mother gave him additional 7
AED. How much money does Abdulla have now?

Solution: List the events on order.


First: Abdulla has 24 AED
Next: He spends 6 AED
Then: He was given additional 7 AED

Therefore, these words do not match the given expression.

Problem 2: Abdulla has 24 AED to spend for his evening snack. He


spends 6 AED for the drinks. Then he bought a hotdog sandwich worth 7
AED. How much money does Abdulla have now?

Solution: List the events on order.


First: Abdulla has 24 AED
Next: He spends 6 AED
Then: He spends 7 AED

Therefore, these words match the given expression.

30
Exerises 1-9:

A- Translate into an expression


1) Lama has 10 pens. She broke 5 pens

2) Hamad has 18 chocolate bars. He ate two bars.

B- Find the pattern. Describe the pattern in words? Then write


the algebraic expression to describe each pattern.

3)

4)

5)

6)

C- Write an expression to match the words.

7) Hamdan shared 10 party freebies equally among his 5 friends.

8) Farook has 12 colored pens. He purchased 5 additional pens.

9) Waleed catches 31 fish. Then he releases 10 fish and catches 6 more.

31
Chapter 1 Lesson 11: Evaluate Numerical
Expressions
GO MATH: Ch.1 L11

Vocabulary Keywords: Learning Outcomes:


Numerical Expression - Recognize the rules in order of
operation operations.
Evaluate -Use the order of operations to
Order of Operations evaluate numerical expressions.

The teacher posted a problem on the board: 20 – 3 x 4. Ali answered 68


while Fady answered 8. Who got the correct answer? It might be
tempting to perform the operations from left to right, and we later get
68 as the answer, that is, 20 – 3 = 17, and then 17 x 4 = 68. But this is
wrong. The correct way to solve this is with the use of Order of
Operations.

Rule: To evaluate, or in other words, to determine the value of a


numerical expression having more than 1 operation, you should follow
strictly the order of operations. Order of Operations guides us on what
to evaluate in a given numerical expression.

Order of Operations

1) Perform the operations within the parenthesis.

2) Multiply and divide from left to right

3) Add and subtract from left to right.

Example 1:Evaluate: 6 x (3 + 6 + 1)

Write the expression 6 x (3 + 6 + 1)

Perform first the operation in 6 x (10)


parentheses

Multiply 60

32
Example 2:Evaluate: 6 + 24 ÷ 8 – 4 x 2

Write the expression 6 + 24 ÷ 8 – 4 x 2

Multiply and divide from left to 6+3-8


right

Add and subtract from left to right. 1

Exercises 1 – 10:

Evaluate the following numerical expressions.

1) (13 +28 - 5 ) ÷ 2 – 3

2) (15 - 4 ) + ( 7 - 20 ÷ 10)

3) ( 9 +33 - 6 ) ÷ ( 10 - 4 )

4) (12 - 3 ) + (12 - 8 ÷ 4 )

5) (12 +17 - 5 ) ÷ 4 – 4

6) 4 x ( 7 x 4 + 6 ) +10

7) ( 8 +48 - 6 ) ÷ ( 23 + 2 )

8) 10 x ( 3 x 7 + 3 ) – 8

9) (17 + 5 ) x (12 - 5 ) – 3

10) (17 + 2 ) x (13 + 3 ) – 4

33
Chapter 1 Lesson 12: Grouping Symbols

GO MATH: Ch.1 L12

Vocabulary Keywords: Learning Outcomes:


- Use order of operations to evaluate
Numerical Expression numerical expressions with
Brackets parentheses, brackets and braces
Braces -Solve word problems involving the
Order of Operations evaluation of numerical expressions.
In the previous lesson, we were able to evaluate numerical expressions
with parentheses. In this lesson, we will deal with more grouping
symbols involving parentheses ( ), brackets [ ] and braces { }.

Rule 1: You can use grouping symbols to write an expression or to group


the operations that go together. To evaluate expressions with grouping
symbols, the expression inside the grouping symbols should be
evaluated first.

Rule 2: Perform operations from the innermost set of grouping symbols


to the outermost pair of numbers. The order in evaluating them is
parenthesis, brackets and then braces.

Example: Perform the indicated operations.

2{ 10[12 + 4(20 - 10) + 30]}

Write the expression. 2{ 10[12 + 4(20 – 10) + 30]}

Perform the operations in the 2{ 10[12 + 4(10) + 30]}


parentheses

Perform the operations in the 2{ 10[12 + 40+ 30]}


brackets
2{ 10[82]}

Perform the operations in the 2{ 820}


braces

Multiply 1,640

34
Exercises 1 – 12:

A- Evaluate the following numerical expressions.

1) { 7[19 + 6(7 + 5) - 14]}

2) 5{ 10[6 + 9(57 - 48) + 12]}

3) 2{ 5[15 + 4(17 - 9) + 9]}

4) 8{ 11[6 + 9(17 + 13) - 52]}

5) 4{ 7[85 - 6(72 - 58) + 21]}

6) 9{ 2[28 + 9(33 - 14) - 28]}

7) 6{ 10[6 + 7(22+ 8) + 10]}

8) 3{ 11[88 -2(6 + 18) -19]}

9) [(16 + 6 ) +(16 ÷ 8 )] + 2

10) [13 +( 8 ÷ 4 - 7 )] + 4

B- Solve.

11) A certain Charity Club produces 35 bars of soap a week and sells
these at $20 each. Before the soap can all be sold, the students
found out that 6 bars were destroyed by mice. How much will be
the total sale at the end of a four-week month? Write an
expression and show your work and answer.

12) Jamila had 30 chocolate cookies to bring to school for her


birthday. Her three classmates wanted two cookies each. Then,
another classmate came to the school that day and he wanted
three cookies. Then, one of her three classmates gave their two
cookies back. Jamila was still passing out cookies. How many
cookies did Jamila have left to pass out after her classmates gave
theirs back?

35
36
C
HAPTER 2
Chapter 2: Divide Whole Numbers

37
Chapter 2 Lesson 1: Place the First Digit

GO MATH: Ch.2 L1

Vocabulary Keywords: Learning Outcomes:


Quotient - Use the place value to place the
Dividend first digit in division.
Divisor Estimate the quotient to place the
Estimate first digit.

When we go to a grocery store and we have a very limited budget to


spend, we normally use estimation on the items that we will buy just to
ensure that your cash in hand is enough. In division, estimation is also
very important to ensure that your final answer would be reasonable.

Rule: We can determine where to place the first digit of a quotient


without dividing them by using basic facts and compatible numbers. We
can estimate the value of the quotient which allows us to place the first
digit of a quotient.

Example: Divide 3,235 ÷ 5. Use place value to place the first digit.

Step 1: Use the place value to place the first digit.

5 3,235 Look at the thousands. 3 thousands cannot be shared


among the 5 groups without regrouping.
Look at the hundreds. 32 hundreds can be shared among 5
groups.
So the first digit is in the hundreds place.

Step 2: Divide the hundreds.

6 Divide: 32 hundreds ÷ 5
5 3,235 Multiply: 6 x 5 hundreds
Subtract: 32 hundreds – 30 hundreds
30
Check: 2 hundreds cannot be shared among 5 groups
2 without regrouping.

38
Step 3:Divide the tens
64 Divide: 23 tens ÷ 5
5 3,235 Multiply: 4 x 5 tens
Subtract: 23 tens – 20 tens
30
Check: 3 tens cannot be shared among 5 groups without
23 regrouping.
20
3

Step 4: Divide the ones.

647 Divide: 35 ones ÷ 5


5 3,235 Multiply: 7 x 5 ones
Subtract: 35 ones – 35 ones
30
23
20
35
35
0

Therefore, 3,235 ÷ 5 = 647

Exercises 1 – 14:Divide

1) 1,054 ÷ 2 2) 4,535 ÷ 5

3) 4,725 ÷ 5 4) 3,375 ÷ 9

5) 2,622 ÷ 3 6) 7,092 ÷ 9

7) 5,208 ÷ 7 8) 1,872 ÷ 4

9) 2,853 ÷ 9 10) 6,300 ÷ 7

11) 1,476 ÷ 6 12) 8,008 ÷ 8

13) 4,008 ÷ 8 14) 7,080 ÷ 3

39
Chapter 2 Lesson 2: Divide by 1-Digit
Divisors
GO MATH: Ch.2 L2

Vocabulary Keywords: Learning Outcomes:


Quotient - Calculate the quotient of 3 – and 4
Dividend – digit dividends and 1 digit
Inverse operation divisors.
Divisor -Test if the quotient is correct using
multiplication.

Mr. Khalid has allotted 120 AED as a budget for the allowance of his 4
kids. How much will each child equally receive? The operation to be
used here is division since the total amount will be shared by each child.
In this lesson, we will deal with division of 3 and 4 digit numbers by a 1
digit number and check the reasonableness of our answers.

Rule: Since multiplication and division are inverse operations, we can


multiply the quotient by the divisor, and if we obtain the given dividend,
then the answer is correct.

Example 1:Chandra’s family is planning a trip to Hong Kong


Disneyland. They will begin their trip in Dubai, UAE, and will travel
5624kilometers over 8 days. If the family travels an equal number
of kilometers every day, how far will they travel each day?
703
Divide: 5,624 ÷ 8 8 5,624
Step 1: Use an estimate to place the first digit 56
in the quotient.
2
0
Estimate: 5,600 ÷ 8 =
700
24
The first digit of the quotient is in the
hundreds place. 24
0

40
Step 2: Divide the hundreds.
Step 3: Divide the tens.
Step 4: Divide the ones
Since 703 is close to the estimate of 700, the
answer is reasonable.
So Chandra’s family will travel 703 kilometers
each day.

Example 2: Divide. Check your answer.

To determine if your answer in a division problem is correct, multiply


the quotient by the divisor. If there is a remainder, add it to the product.
The result should be equal the dividend.

102 r3 102 ← qoutient


6 615 × 6 ← divisor
−6 612
01 + 3 ← remainder
−0 615 ← dividend
15
− 12
3
Since the result of the check is equal to the dividend, the division is
correct.
So, 615 ÷ 6 is 102 r3 .

Exercises 1 – 10:Divide and check your answer.

1) 2,866 ÷ 7 2) 6,888 ÷ 8

3) 1,552 ÷ 8 4) 1,393 ÷ 6

5) 3,138 ÷ 3 6) 5,039 ÷ 8

7) 6,365 ÷ 2 8) 1,858 ÷ 6

9) 8,397 ÷ 2 10) 3,660 ÷ 9

41
Chapter 2 Lesson 3: Division with 2-Digit
Divisors
GO MATH: Ch.2 L3

Vocabulary Keywords: Learning Outcomes:


Partial Quotient - Use base – ten blocks to model to
Dividend understand division of whole numbers
Divisor -Compute the quotient of numbers and
1 – digit numbers using models.

Suppose there are 209 students in a certain school. If these students will
be assigned in 11 community projects, how many students are there in
each group? In this situation, we can use division to find out the number
of students per group. In this lesson, let us try to use models to
investigate division.
Rule: To interpret division of whole numbers, use base – ten blocks. Use
hundreds, tens and ones blocks in order to model the dividend. Then
Use one hundred block and ten blocks to show 10 groups having the
same number as the divisor. Use the other blocks to make more groups
of the same size. Sometimes, you may need to regroup hundreds or tens.
Example: Divide 154 by 11.
Step 1: Model 154 with base – ten blocks.

Step 2: Make equal groups of 11. Each group should contain 1 ten and 1
one. We can make 4 groups of 11 without regrouping.

42
Step 3: Regroup 1 hundred as 10 tens. Regroup 1 ten as 10 ones.

Step 4: Use the regrouped blocks to make as many groups of 11 as


possible. After that, count the total number of groups.

There are 14 groups. Therefore, the quotient is 14.

Exercises 1 - 8:

A- Divide using base – ten blocks.

1) 112 ÷ 8 2) 135 ÷ 9

3) 72 ÷ 4 4) 120 ÷20

5) 112 ÷ 7 6) 114 ÷6

B- Solve.

7) Madhat baked 84 blueberry muffins. If each muffin tray held 3


muffins, how many trays did Madhat use?
8) There are 156 students in a school’s Mathematics Club. How many
groups of 12 can the students make?

43
Chapter 2 Lesson 4: Partial Quotients

GO MATH: Ch.2 L4

Vocabulary Keywords: Learning Outcomes:


Quotient -Recognize partial products.
Dividend -Use partial quotients to divide 2 –
Partial Quotient digit divisors.
Divisor

A pizza restaurant recorded that their customers eat about 24 pounds of


pizza per person every year. If you ate that much pizza each year, how
many years would it take you to eat 875 pounds of pizza?

We can use the long division to solve this problem. But this time, let us
try a different method that is, using the partial quotients.

Rule: To use partial quotients to divide numbers by 2 – digit divisor,


multiply the divisor by 10 and keep subtracting that product from the
given dividend until you come up with a result that is less than the
product. After this, subtract other multiples of the divisor until the result
is less than the divisor or 0.

Example 1:Referring to the problem above, Divide 875 by 24 using


partial quotients.

775 ÷ 23

Step 1: Step 2:

Subtract multiples of the divisor Subtract smaller multiplies of


from the dividend until the the divisor until the
remaining number is less than the remaining number is less than
multiple. The easiest partial the divisor. Then add the
quotients to use are multiples of partial quotients to find the
10. quotient.

44
24 875
− 240 10 × 24 10
635
− 240 10 × 24; 10
395
− 240 10 × 24; 10
155
− 120 5 × 24; 5
35
− 24 1 × 24; +1
11 36

875 ÷ 24 is 36 r 11

So, it would take you more than 36 years to eat 875 pounds of pizza.

Exercises 1- 8:

A- Divide using partial products.

1) 1,127 ÷ 22 2) 1,715 ÷ 11

3) 9,822 ÷ 28 4) 1,732 ÷ 52

5) 6,932 ÷ 29 6) 4,734 ÷ 45

B- Solve.

7) A city has 8,028 AED to buy new street signs. If each sign costs
18AED, how many new street signs will the county be able to buy?

8) Monir's Crackers will make 8,738 ounces of cheese crackers next


year. The company plans to put the crackers into 17-ounce boxes.
How many boxes will the company be able to fill next year?

45
Chapter 2 Lesson 5: Estimate with
2-Digit Divisors
GO MATH: Ch.2 L5

Vocabulary Keywords: Learning Outcomes:


Quotient - Use compatible numbers to estimate
Dividend quotients.
Compatible Numbers -Solve problems involving estimation
Divisor with 2 digit divisors.

Suppose Farhan wants to divide his one - month allowance worth


1,550AED to 14 street children, by approximately how much will each
child receive?

In some situations, finding the estimated amount is more appropriate to


do than computing the exact amount. In the problem above, it will not be
necessary to give the exact division of money since there are only few
denominations of coins. In this lesson, we are going to use compatible
numbers in order to estimate quotients.

Rule: To use compatible numbers to estimate quotients, round the


divisor to any multiple of 10 and round also the dividend to the nearest
multiple of the new divisor. Lastly, use the basic division fact pattern of
0s to complete the estimation.

Example: The Burj Khalifa, the tallest building in the world has a
height of approximately 2,717 feet. This building has 160 stories. If
elevators lift the visitors to the 80th floor of the building in 60
minutes, about how many feet do the elevators travel per second?

Estimate. 2,717 ÷ 60

Step 1: Use two sets of compatible numbers to find two different


estimates.

2,717 ÷ 60  2,400 ÷ 60 2,717 ÷ 60  3,000 ÷ 60

46
Step 2: Use patterns and basic facts to help estimate.

24 ÷ 6 = 4 30 ÷ 6 = 5

240 ÷ 60 = 4 300 ÷ 60 = 5

2,400 ÷ 60 = 40 3,000÷ 60 = 50

The elevators travel about 40 to 50 feet per second. The more


reasonable estimate is 3,000÷ 60 = 50 since 3,000 is closer to 2,717
compared to 2,400.

Therefore, the elevators travel about 50 feet per second.

Exercises 1 – 10:

A- Use compatible numbers to find two estimates.

1) 569 ÷ 12 2) 3,576÷ 34

3) 7,464 ÷ 23 4) 9,876 ÷ 17

5) 9,755 ÷ 53 6) 6,853 ÷45

7) 3,475 ÷ 97 8) 8,763 ÷22

B- Solve.

9) A carpenter has 7,866AED to buy wood. If each piece of wood


costs 5AED, about how many pieces can the carpenter buy?
Choose the better estimate.

10) A group of 5,175 businessmen needs to take buses to the airport.


If each bus can take 4 men, about how many buses will the
business people need? Choose the better estimate.

47
Chapter 2 Lesson 6: Divide by 2-Digit
Divisors
GO MATH: Ch.2 L6

Vocabulary Keywords: Learning Outcomes:


Quotient - Calculate the quotient when two
Divisor numbers are divided by one
Dividend another.
Remainder -Recognize the remainder in
division

Mr. Ibrahim has allotted 1,200 AED as a budget for his 4 charitable
institutions he is helping. How much will each institution equally
receive? The operation to be used here is division since the total amount
will be shared by each charitable institution. In this lesson, we will deal
similar problems and check the reasonableness of our answers.

Example 1:Divide. 4,603 ÷ 30 = n


Step 1:
Decide where to place the first digit
in the quotient. Are there enough
thousands? NO; 5 < 30. Are there
enough hundreds? YES; 56 > 30.

The first digit goes in the hundreds place.

Step 2:
Divide the hundreds.
Write the 1 in the hundreds place.
Multiply. 30 × 1
Subtract. 46 − 30
Compare. 16 < 30

Step 3:
Divide the tens.
Write the 5 in the tens place.
Multiply. 30 × 5
Subtract. 160 − 150
Compare. 10 < 30

48
Step 4:
Divide the ones.
Write the 3 in the ones place.

Multiply. 30 × 3
Subtract. 103 − 90
Compare. 13 < 30

So, 4,603 ÷ 30 = 153 r 13

Example 2:Bright Video Shop received a shipment of 832 video


cassettes. The video cassettes were packaged in 32 cartons. Each
carton held the same number of cassettes. How many video
cassettes were in each carton?

Step 1:832 ÷ 32
Decide where to place the first digit.
Are there enough hundreds?
NO; 8 < 32.
Place the first digit in the tens place.

Step 2:
Divide the 83 tens.
Multiply. 32 × 2
Subtract. 83 − 64
Compare. 19 < 32

Step 3:
Divide the 192 ones.
Multiply 32 × 6
Subtract 192 − 192

So, each carton held 26 video cassettes.

You can use multiplication to check your answer. Multiply the divisor by
the quotient. Add any remainder.
32 × 26 = 832. The answer checks.
49
Example 3:Ali watches TV for 28 hours every week. How many
hours does he watch per day?

28 ÷ 7 = 4 hours per day

Exercises 1-12: Divide.

1) 3,672 ÷ 40 2) 4,567 ÷ 60

3) 2,567 ÷ 30 4) 1,873 ÷ 20

5) 1,463 ÷ 50 6) 2,879 ÷ 30

7) 1,872÷ 24 8) 1, 440 ÷32

9) 1,392÷16 10) 1,242÷ 23

11) 2,880 ÷ 64 12) 360 ÷15

50
Chapter 2 Lesson 7: Interpret the
Remainder
GO MATH: Ch.2 L7

Vocabulary Learning Outcomes:


Keywords: - Interpret the remainder in
Quotient dividing numbers.
Dividend -Solve word problems involving
Remainder quotients with remainder.

When there is a remainder in a division problem, you need to look at the


question to see what is being asked. For instance, If you have 100 AED
and want to divide your money equally to 3 savings accounts, you may
have 30 AED in each savings account and you have excess money worth
10 AED. This excess is the remainder.

You may drop the remainder, or round the quotient to the next greater
whole number, or you may use the remainder as a fraction part of your
answer.

Rule: When solving the division problem, the remainder should be


expressed as a fraction in case the remainder needs to be part of the
quotient. The situation that is described in the problem will determine if
the remainder must be written in fraction form.

Example 1:Amna made punch with 45 ounces of orange juice, 32


ounces of pineapple juice, and 54 ounces of soda. How many 6-
ounceservings did she make?

45 + 32 + 54 = 131 oz

There are 5 ounces left over.


That is not enough for another
6-ounce serving. Drop the remainder.

So, Amna made 21 six-ounce servings.

51
Example 2:Tell how you would interpret the remainder. Then give
the answer?

Hana has 445 books in her library. She needs to pack them in boxes.
Each box has a capacity of 20 books. How many boxes does Hana need?

22 r5
20 445
40
45
40
5

Hana needs 22 boxes. There are 5 books left out.

Exercises 1 – 5: Solve

1) A total of 126 players are riding in cars to the soccer games.


If 5 players can ride in each car, how many cars are needed?

2) There are 479 books in the library, on each shelf there are 9 books.
How many shelves are needed to put all the books?

3) Books are on sale for 7 Dirhams. Ibram has 30AED in his wallet. How
many books can he buy?

4) There are 32 students in a 6h grade class. Each table in the classroom


seats 6 students. How many tables will be needed?

5) Nawaz charges an hourly rate for babysitting. During the summer


vacation she works 8 hours babysitting and earns $84.00. How much
does Nawaz charge per hour?

52
Chapter 2 Lesson 8: Adjust Quotients

GO MATH: Ch.2 L8

Vocabulary Keywords: Learning Outcomes:


Quotient - Recognize the need to adjust the
Dividend quotient if the estimate is too high
Divisor or too low.
Estimate -Solve word problems involving
division.

In lesson 3, we learned how to estimate quotients. We found out that


there are two estimates on the speed of the elevator of the Burj Khalifa
to be 40 and 50 feet per second but we have chosen the 50 to be the best
estimate since this is the closest.

We can adjust the quotient in case our estimate is too high or too low.

Rule 1: In case the product is higher than the dividend, decrease the
estimate by 1. In case the difference is higher than or equal to the
divisor, then we have to increase the estimate by 1.

Rule 2: If you get the quotient of two numbers, you can use the first digit
of your estimate as the first digit of your quotient. There are cases
where the first digit will be very high or very low. In this situation,
adjust the quotient by increasing or increasing he first digit.

Example 1:Divide 372 by 45. Estimate 300 ÷ 50 = 6

Try 6 ones.
6
45 372
-270
102
Since 102 is greater than the divisor 45, then the estimate is too low.

53
Example 2:Divide 2,461 by 27. Estimate 3,000 ÷ 30 = 100

10
27 2,461
-2,700

Since we cannot subtract 2,700 from 2,461, then the estimate is too high.

Exercises 1 – 9:

A- Adjust the estimated digit in the quotient, if needed and then


divide.

1) 865 ÷ 12 2) 1,446 ÷ 15

3) 8,543 ÷ 42 4) 2,545 ÷26

5) 1,248 ÷ 16 6) 7,856 ÷ 24

B- Solve.

7) A teacher has 288 pieces of chocolates. If there are 36 students, 6


of whom are boys, if divided evenly, how many pieces of
chocolates will each student get?

8) The park's sprinklers can spray 965 gallons of water on the grass
in 24 minutes. How many gallons can they spray in one minute?

9) Mandal sold 504 doughnuts for her school fundraising campaign.


The doughnuts were sold in boxes with two dozen doughnuts
each. How many boxes did she sell?

54
Chapter 2 Lesson 9: Problem Solving:
Division
GO MATH: Ch.2 L9

Vocabulary Keywords: Learning Outcomes:


Quotient - Draw a diagram to solve division
Dividend problems.
Divisor -Calculate division represented by
word problems.

Beef cattle are cattle that are raised for beef production. If a full grown
cattle is weighing 625 pounds, how many beef packages can be made if
each pack weighs 25 pounds? To solve this problem, drawing a diagram
might be useful. In this lesson we will use the strategy “draw a diagram”
to help us solve problems easily.

Rule: Making a diagram like bar models or area models helps you
organize the information in the given problem.

Example: Mousa and his family chartered a fishing boat for the day.
Mousa caught a mackerel and a milkfish. The weight of the milkfish
was 7 times as great as the weight of the mackerel. The combined
weight of both fish was 512 pounds. How much did each fish weigh?

Step 1: Identify what you need to find.


We need to find the weight of each fish.

Step 2: Determine what information you need to use.


We need to use the total weight of the two fish that is 512 pounds
and the fact that the milkfish is 7 times as heavy as the mackerel.

Step 3: Determine how you will use the information.


Use the strategy “draw a diagram” and then divide. Draw a bar
model to show division to determine the weight of each fish.

Step 4: Solve the Problem

Draw one box to show the weight of the mackerel. Then draw a
bar of 8 boxes of the same size to show the weight of the milkfish.
55
Then divide the total weight of the two fish by the total number of
the boxes.

mackerel 64
512 pounds
milkfish 64 64 64 64 64 64 64

64
8 512
- 512
0

So, the mackerel weighed 64 pounds and the milkfish weighed 448
pounds.

Exercises 1 – 3:

1) Sarah and Monir picked strawberries on the whole weekend.


Monir picked 9 times as many strawberries as Sarah. Together,
they harvested 310 strawberries. How many strawberries did
each person harvest?

2) Raed harvested 11 times green mangoes as Raja. Together, they


harvested 936 green mangoes. How many green mangoes did each
person pick?

3) Nadaa wrote 10 times as many pages of a school article as Noor.


They wrote 396 pages all in all. How many pages did each student
write?

56
C
HAPTER
Chapter 3: Add and Subtract
3
Decimals

57
Chapter 3 Lesson 1: Thousandths

GO MATH: Ch.3 L1

Vocabulary Keywords: Learning Outcomes:


Thousandths - Recognize the meaning of
Place Value thousandths.
- Model decimals to thousandths.

In machines, measurements are in


thousandths of an inch. If one piece is not
measured accurately it will fail to work
properly. A few thousandths of an inch can
ruin an engine. Adjustments of engines (i.e.
spark plug gapping) is measured in
thousandths of an inch.

Thousandths are obtained by dividing hundredths by ten equal parts.

Rule: The value of one place – value position is equal to ten times as
much as the value of the position to the right while it is 1/10of the value
of the position to its left.

Decimal
Ones Tenths Hundredths Thousandths
Point

• ? 0.08 ?

10 times 1/10 of as much

0 8 is 10 times as much as 0.08 while 0.008 is 1/10 of 0.08

58
Example: Complete the table below.

Step 1: Write the given decimal in the place value table

Step 2: Write a decimal that is 10 times as much as the given decimal


using the place value table

Step 3: Write a decimal that is 1/10 of the given decimal.

10 times
Decimal as much 1/10 of
as

0.06 0.6 0.006

0.2 2.0 0.02

0.09 0.9 0.009

Exercises 1 – 5:

Use the place value patterns to fill the table.

10 times
Decimal as much 1/10 of
as

1) 0.3

2) 0.07

3) 0.05

4) 0.4

5) 0.1

59
Chapter 3 Lesson 2: Place Value of Decimals

GO MATH: Ch.3 L2

Vocabulary Learning Outcomes:


Keywords: -Identify the place value of a digit in
Decimal number a decimal number
Place value -Write a decimal number in
Decimal point standard form, word form or
expanded form

In swimming competitions, the times the swimmer finished are always


given in decimals to accurately determine the winner of the said
competition. This is how important the decimals are. In this lesson, we
will learn about the place value of decimals and write them in different
forms.

Base-ten blocks can be used to model decimals.

A square represents The whole is into 100


The whole is divided
divided one whole, or equal parts.
into 10 equal parts.
1

1
Each part is of a whole or 0.1.
10
1
Each part is of a whole, or 0.01.
100

one one tenth one hundredth


1.0 0.1 0.01
1 1
1
10 100

60
Example 1:Find the value of the underlined digit.

1) 1.023 2) 0.945 3) 24.821

3 9 8
1000 10 10
0.003 0.9 0.8

4) 5.761 5) 9.391

6 1
100 1000
0.06 0.001

A place-value chart can help you find the value of each digit in adecimal.

Ones . Tenths Hundredths Thousandths Ten


Thousandths
3 . 2 5 6 1

Decimals:

Read: Three Two Five Six 0ne ten


tenths hundredths thousands thousandths

Write: 3 0.2 0.05 0.006 0.001

Standard Form:3.2561

Expanded Form: 3.0 + 0.2 + 0.05 + 0.006 + 0.0001

Word Form: three and two thousand, five hundred fifty-one ten
thousandths.

61
Example 2:Write each number in two other forms.
1) 7.0432
7.0 + 0.04 + 0.003 + 0.0002
Seven and four hundred thirty-two ten-thousandths

2) Five and three thousand, nine hundred eighty-one ten-


thousandths
5.3981
5 + 0.3 + 0.09 + 0.008 + 0.0001

3) 9.0 + 0.4 + 0.06 + 0.007 + 0.0005


9.4675
Nine and four thousand, six hundred seventy-five ten-thousandths

Example 3:The silk spun by a spider in Mr. Abdul’s backyard is


about 0.002 millimeter thick. A commonly used sewing thread is
approximately 0.2 millimeter thick. Compare the thickness of the
spider’s silk and the thread.

Step 1: Write the numbers in a place-value chart.

Step 2: Count the number of decimal place-value positions to the digit 3


in 0.2 and 0.002.

0.2 has 2 fewer decimals places than 0.002.


2 fewer decimal places:10 × 10 =100
0.2 is 100 times as much as 0.002.
1
0.002 is of 0.2
100

Therefore, the thread is 100 times as thick as the spider’s silk .the
1
thickness of the spider’s silk is that of the thread.
100

62
Exercises 1-16:

A- Write each number in two other forms


1) 8,1234

2) 4.5673

3) 5 + 0.1 + 0.02 + 0.008 + 0.0005

B- Find the value of the underline digit


4) 2.135

5) 1.657

6) 3.975

7) 9.347

8) 8.583

9) 4.531

C- Write a decimal point in standard form.

10) seven thousandths

11) 0.7 + 0.04 + 0.002

12) four and 6 tenths

13) two and thirteen hundredths

14) 0.2 + 0.002

15) seventeen thousandths

D- Solve:

16) In an investigatory project, Mustafa found two sizes of


cockroaches. One is 1.452 and the other one is 1.5 inches long. Use
the place value chart to determine which one has a longer length.
63
Chapter 3 Lesson 3: Compare and Order
Decimals
GO MATH: Ch.3 L3

Vocabulary Keywords: Learning Outcomes:


Decimal -Recognize the comparison
Order symbols.
Place value - Use the place value to
Compare compare and order decimals.

In a big festival at Town ABC, there is one part of the event where the
contest is about the heaviest hog alive in their town. The weights of the
hogs of Noura, Gamal, Kumar and Saeed are 252.09 lbs, 247.99 lbs, 252.8
lbs and 236.9 lbs, respectively. Whose farmer’s hog weighed the most?
To determine the winner of this event, it would be necessary to compare
the weights to find out whose hog is the heaviest.

One way to compare decimals is to use the place value chart.

Rule: To use the place value chart in comparing and ordering decimals,
align the decimal points of the given numbers and compare and order
them. Compare the digits in each place value position from greatest to
least value. The digit that is greatest is in the greatest number. In case
the digits are equal, move to the next lesser place value and compare the
corresponding digits.

Example 1:The table lists some of the mountains in the United Arab
Emirates that are over one kilometer high. How does the height of
Jabal Yibir in Ras Al Khaimah compare to the height of Jabal Qitab
in Al Fujairah? Order the heights from least to greatest. Which
mountain has the least height? Which has the greatest height?

Mountain Heights
Height (in
Mountain
kilometers)
Jabal Bil ‘Ays 1.934
Jabal Yibir, Ras Al Khaimah 1.527
Jabal Qitab, Jabal Qitab 1.029

64
Align the decimal points. Start from left to right. Compare digits in each
place-value position until the digits are not the same.

Step1: Compare the Step 2: Compare the


ones. tenths.
1.527 1.527
↓ 1=1 ↓ 5>0
1.029 1.029

Since 5 > 0 , then

Therefore, the height of the Jabal Yibir is greater than the height of
the Jabal Qitab.

Compare the height of Jabal Yibir to Jabal Bil ‘Ays.

=1 1 0< 9

Since 0 < 9 , then 1.029 < 1.934

Therefore, the height of Jabal Yibir is less than the height of Jabal Bil
‘Ays

From our computation above, we found out that the height of the Jabal
Yibir is greater than the height of the Jabal Qitab and the height of
Jabal Yibir is less than the height of Jabal Bil ‘Ays. Therefore, arranging
the heights from least to greatest, we have, 1.029, 1.527, 1.934. This
implies that the Jabal Qitab has the least height and the Jabal Bil ‘Ays is
the highest.

65
Exercises 1 – 16:

A- Compare each pair of decimals using a <, > or = sign using the
place value chart.

1) 17.967 _____ 17.69 2) 13.255 _____13.525

3) 18.797 _____ 18.866 4) 12.382 _____ 12.638

5) 7.122 _____ 6.908 6) 19.367_____ 19.701

7) 11.324_____ 11.456 8) 9.849 _____9.494

9) 1.257 _____ 1.364 10) 12.997_____ 12.86

B- Order the following decimals using the place value chart.

11) 0.292, 0.929, 0.834, 0.334, 0.682, 0.122, 0.774, 0.123, 0.987,

(Least to Greatest)

12) 0.009, 0.888, 0.885, 0.611, 0.916, 0.157, 0.654, 0.93, 0.015, 0.867

(Greatest to Least)

13) 1.338, 1.729, 1.755, 1.153, 1.806, 1.993, 1.45, 1.152, 1.935, 1.095

(Least to Greatest)

14) 1.048, 0.929, 0.871, 1.097, 0.828, 0.92, 1.073, 1.061, 1.162, 1.066

(Greatest to Least)

15) 2.456, 2.341, 2.368, 2.326, 2.369, 2.474, 2.413, 2.486, 2.352

(Greatest to Least)

C- Solve.

16) Ahmad is setting out four boards of lumber. The lengths of the
boards are 4.5 feet, 4.52 feet, 4 feet, and 4.505 feet. Order the
lengths from longest to shortest.

66
Chapter 3 Lesson 4: Round Decimals

GO MATH: Ch.3 L4

Vocabulary Keywords: Learning Outcomes:


Decimal -Recognize the place value of the
Round digits in a decimal.
Place value Use place value to round decimals
to a given place.

In the previous lessons on whole numbers, we use base ten numerals to


round off whole numbers to estimate our quotients. Recall that we use
an estimated value of 3,000 to estimate the height of the Burj Khalifa
whose height is 2,717 feet. In this lesson, we will also round off numbers
to a given place value, but this time, involving decimals.

Rule: To round decimals to a given place, find the digit in the place to
which you are rounding. Next, check the digit at its right. If the digit at
the right is greater than or equal to 5, then add 1 to the digit in the place
you are rounding. If the digit to the right is less than 5, do not change the
digit you are rounding then drop all the digits to its right.

Example 1:The smallest frog in the world found in America is


approximately 0.387 of an inch long. Round off this length to the nearest
hundredths of an inch.

• Write the decimal in a place-value chart and encircle


the digit in the place value to which you want to round.

• In the place-value chart,


underline the digit at its right to
which you are rounding.

• Since the digit to the right of 8 is 7 which is greater than 5, then


digit in the rounding palace increases by 1.

67
• Drop the digit after the place to which you are rounding, so we have
to drop 7.

Therefore, to the nearest hundredths of an inch, the frog is about


0.39 of an inch long.

Example 2:The smallest known spider is Patumarplesi (family


Symphytognathidae) of Western Samoa. It measured 0.017 in in
length. What is the length of the smallest spider to the nearest
hundredths of an inch?
0.017
↓ 7>5
0.02

So, the nearest hundredths of an inch, the frog is about 0.02 of an inch
long.

Exercises 1 – 12:

A- Round each number to the nearest hundredth.

1) 2.345 2) 6.232

3) 7.811 4) 8.556

5) 3.236 6) 8.812

7) 7.5521 8) 1.7912

9) 6.3988 10) 5.3638

B- Solve.

11) Find the decimal that represents 0.7168 rounded to the nearest
hundredth and then to the nearest tenth.

12) Express 298.3669 rounded to the nearest hundredth and then


to the nearest tenth.

68
Chapter 3 Lesson 5: Decimal Addition

GO MATH: Ch.3 L5

Vocabulary Keywords: Learning Outcomes:


Decimal Addition -Model decimal addition using
Sum base – ten blocks.
-Compute the sum of decimals.

Suppose you purchased some items in Westzone and the prices are as
follows: 1.25 AED, 23.24 AED, 2,55 AED and 3.89 AED. What is the total
amount of money that the cashier will take from your debit card?

To find the sum of decimals, one way is to use base – ten blocks as
shown below.

1 0.1 0.01
Rule: To use the base – ten blocks, model each addend using flat, longs
and small cubes. Add the hundredths and if needed, regroup each 10
hundredths as 1 tenth. Add the tenths and if needed, regroup each 10
tenths as 1 one.
Example: Add 2.5 and 2.7
Step 1: Model the sum using base – ten blocks.

2 2 0.5 (0.5 + 0.2 = 0.7)

69
Step 2: Add the tenths. Since there are more than 9 tenths, regroup.

Step 3: Redraw the figure then record the sum.

Therefore, 2.5 +2.7 = 5.2

Exercises 1 – 5:

Add the following decimals using base – ten blocks.

1) 0.8 + 2.1

2) 3.1 + 1.5

3) 2.7 + 1.8

4) 2.7 + 3.8

5) 1.9 + 4.2

70
Chapter 3 Lesson 6: Decimal Subtraction

GO MATH: Ch.3 L6

Vocabulary Keywords: Learning Outcomes:


-Model decimal subtraction using
Decimal Subtraction base – ten blocks.
Difference -Compute the difference of
decimals.

In the previous lesson, we used the base ten blocks to add decimals. This
time, we will use these blocks in order to model subtraction.

Rule: In using base ten blocks, model the first number using flats, longs
and small cubes. After this, model the number being subtracted by
removing the blocks that represent the number. If required, do
regrouping.

Example: Subtract 3.82 – 1.47

Step 1: Model 3.82 – 1.47 using base ten blocks. Subtract the
hundredths. Since there are no enough hundredths, regroup.

Step 2: Subtract the tenths. Since there are no enough tenths, we have to
regroup. Subtract the ones.

71
Step 3: Redraw the models and record the difference.

So, 3.82 – 1.47 = 2.35

Exercises 1 – 10:

Subtract the following using base – ten blocks.

1) 0.86 – 0.42

2) 1.23 – 1.02

3) 2.29 - 0.38

4) 2.68 – 0.58

5) 3.14 – 0.67

6) 2.54 – 1.75

7) 1.51 – 1.27

8) 3.84 – 1.59

9) 3.43 – 2.14

10) 3.62 – 1.18

72
Chapter 3 Lesson 7: Estimate Decimal Sums
and Differences
GO MATH: Ch.3 L7

Vocabulary Keywords: Learning Outcomes:


-Estimate decimal sums.
Benchmark -Estimate decimal
Sum differences.
Difference

In lesson 5, we learned how to round decimals using base – ten models.


In this lesson we will learn how to use rounding in estimating sums and
differences of decimals.

Rule: Rounding off decimals and using the benchmarks on a number line
can be used to get an estimated value of sums and differences.

Definition: The common numbers that are used as reference points are
called benchmarks. The common benchmarks are 0, 0.25, 0.50, 0.75
and 1.

Example 1:Use rounding to estimate. Round to the nearest whole


dirham then subtract.
28.67 ⇒ 29 Add 1 to 8 since 6 > 5
−12.18 ⇒ −12 Keep 12 then drop .18
17

Therefore, to the nearest whole Dirham, 28.67AED – 12.18AED is about


17 AED.

73
Example 2:Use benchmarks to estimate 0.19 + 0.45

Plot the decimals on a number line with marked benchmarks.

0.19 is closer to 0.25 and 0.45 is closer to the benchmark 0.5. So the
estimate sum is 0.25 + 0.50 = 0.75.

Exercises 1 – 10:

A- Use rounding to estimate the sum or difference.

1) 28.83 +87.15

2) 82.14 - 47.47

3) 43.42 - 38.83

4) 49.19 - 45.45

5) 87.83 +56.54

B- Use benchmarks to estimate the sum or difference.

6) 57.19 - 48.61

7) 89.83 +56.29

8) 93.67 +73.36

9) 37.54 - 10.61

10) 60.38 - 19.67

74
Chapter 3 Lesson 8: Add Decimals

GO MATH: Ch.3 L8

Vocabulary Keywords: Learning Outcomes:

Sum -Add decimals using place value.


Equivalent Decimals -Estimate the sum of decimals.

Machine operators, manufacturing and engine


professionals use an outside micrometer in order to
measure the outside diameter of a cylindrical or
spherical object to one ten-thousandth of an inch
(.0001). Suppose you measured the thickness of two tiny objects using
this micrometer. How are we going to determine the sum of those
measures in decimals? In lesson 5, we were able to find the sum of
decimals using models. This time, we will use the place value.

Rule: When adding two or more decimals, use equivalent decimals to


ensure the alignment of the digits in each place in the place value chart.
To produce equivalent decimals, we can add as many zeros as needed
after the last digit of the decimal part so that the addends will have the
same number of decimal places. For instance, 1.2 can be written as 1.20.

Example 1:Deepak is travelling back to his hometown and the night


before his flight, he measured his 2 hand carry bags to make sure
that it will not exceed 7 kg. The weights of his two bags are 2.38
1.81 kilograms. What is the total weight of the two bags?

• Add the hundredths first.


8 hundredths + 1 hundredths = 9 hundredths 1
2.38
• Then add the tenths and ones. Regroup as needed.
+1.81
3 tenths + 8 tenths = 11 tenths. Regroup.
4.19
2 ones + 1 one + 1 regrouped one = 4 ones.
• Record the sum for each value places.

Therefore, the sum is 4.19


75
Example 2:Estimate. Then find the sum of 20.3 + 13.87.

Step 1: Estimate the sum. Step 2: Find the sum.

Add the hundredths first.

Then, add the tenths, ones, and, tens.

Regroup as needed.

20.30 + 13.87 = 34.17

Exercises 1 – 10: Estimate then find the sum.

1) 90.14 + 65.42

2) 90.21 + 77.51

3) 44.38 +83.12

4) 94.73 +90.89

5) 23.36 +23.97

6) 83.86 + 24.19

7) 54.84 +97.73

8) 52.25 + 48.74

9) 80.41+ 69.54

10) 14.22 +53.62

76
Chapter 3 Lesson 9: Subtract Decimals

GO MATH: Ch.3 L9

Vocabulary Keywords: Learning Outcomes:


-Use place value to subtract
Difference decimals.
Subtract -Solve word problems involving
decimal subtraction.

We can use estimation and place value to solve problems involving


subtraction of decimals like receiving change, finding fractional
distances, salary deduction, etc.

Rule 1: To subtract using place value, line up the place values then
subtract according to the place values of the digits, that is, hundredths to
hundredths, tenths to tenths and ones to ones, etc.

Example 1:Noora has 3.37 kilograms of grapefruit and 2.25


kilograms of star fruit. She estimates that she has about 1 more
kilogram of grapefruit than star fruit. How many more kilograms of
grapefruit than star fruit does Noora have? How can you use this
estimate to decide if your answer is reasonable?

Subtract. 3.37 − 2.25


• Subtract the hundredths first.
7 hundredths - 5 hundredths = 2 hundredths

• Then subtract the tenths and ones.


3 tenths - 2 tenths = 1 tenths
3 ones - 2 ones = 1 one
• Record the difference for each place value.

So, Noora has 1.12 more kilograms of grapefruit than star fruit.
Since1.12 is close to 1, the answer is reasonable.

Rule 2:Since subtraction and addition are inverse operations, you can
check subtraction by adding.

77
Example 2:Find the difference of 14.3 - 8.64. Use addition to check.

Step 1: Find the difference. Step 2: check your answer.

Subtract the hundredths first. Add the difference to the number you
subtracted. If the sum is equal to the
Next, subtract the tenths, ones,
number you subtracted from, your
and tens. Regroup as needed.
answer is correct.
0 13 12 10
14.30 11
- 8.64 5.66 ← difference
5.66 + 8.64 ← number subtracted
14.30 ← number subtracted from

Example 3:Hala has 106.87 Aed. She pays 56.75 Aed for a book that
she bought from the bookshop. How much money does she still
have?

106.87 – 56.75 =?

106.87
- 56.75
50.12 Hala is left with 50.12 Aed.

Exercises 1-10:

A- Find the difference using place value. Check your answer


using addition.

1) 34.25 - 6.37 2) 112.37 - 98.56

3) 12.54 – 10.21 4) 79.34 – 28.13

5) 78.9 - 23.4 6) 156.79 – 141.23

7) 135.67 – 112.13 8) 92.87 - 21.63

78
B- Solve.

9) It rained 0.25 inches on Sunday. On Monday, it rained 0.06 inches less


than on Sunday. How much did it rain on Monday?

10) Akram weighed two colored metal balls during a science class. The
yellow ball weighed 0.9 pounds and the red ball weighed 0.6 pounds.
If Akram places both balls on the scale at the same time, what will the
scale read?

79
Chapter 3 Lesson 10: Patterns with
Decimals
GO MATH: Ch.3 L10

Vocabulary Keywords: Learning Outcomes:


-Use addition or subtraction to
Sequence describe a numerical pattern.
Term Write the next terms in a
sequence of numbers.

Musical elements like beat, rhythm, melody, and tempo


have mathematical principles like spatial properties,
sequencing, counting, patterning, and one-to-one
correspondence. In this lesson, we are going to learn about
patterns focusing on patterns with decimals.

Definition 1: A list of numbers arranged in order is called a sequence.

Definition 2: The numbers in the sequence are called terms.

Example 1:A park rents kayaks for visitors to use at


the river. It costs 25.00AED to rent a kayak for 1
hour, 26.75AED for 2 hours, 28.50AED for 3 hours,
and 30.25AED for 4 hours. If this pattern continues,
how much should it cost Ali to rent a kayak for 7
hours?

Step 1:

Write the terms you know in a sequence. Then look for a pattern by
getting the difference from one term in the sequence to the next.

Step 2: Write a rule that describe the pattern in the sequence.

Rule: add 1.75AED

80
Step 3: Extend the sequence to solve the problem.

25.00, 26.75, 28.50, 30.25, 32.00 , 33.75 , 35.50

So, it should cost 35.50 AED to rent a kayak for 7 hours.

Example 2:Write a rule for the pattern in the sequence. Then find
the unknown terms in the sequence.

49.6, 48.3, 47, 45.7, 44.4 , 43.1 , 41.8 , 40.5, 39.2

Step 1: Look at the first few terms in the sequence.


Check if the sequence increasing or decreasing from one term to
the next.

Step 2: Write a rule that describes the pattern in the given sequence.
Note that the operation that can be used to describe a sequence
that increases is addition .

Note that the operation that can be used to describe a sequence


that decreases is subtraction .
Rule: subtract 1.3 .

Step 3: Use your rule to find the unknown terms.Then complete the
sequence above.

Exercises 1- 8:

Write a rule for the following sequences. Then write the next three
terms or missing terms.

1) 1.5, 2.0, 2.5, 3.0,___,___,___ 2) 0.4, 0.8, 1.2, 1.6,___,___,___

3) 4.4, 5.4, 6.4, 7.4,___,___,___ 4) 2.3, 2.6, 2.9, 3.2,___,___,___

5) 15.9, 16.2, 16.5, 16.8,___,___,___ 6) 18.5, 18.0, 17.5, 17.0,___,___,___

7) 5.9, ____, 5.5, 5.3, ____4.9, 4.7 8) 3.3, 3.6, ____, ____ ,4.5, 4.8, 5.1

81
Chapter 3 Lesson 11: Problem Solving: Add
and Subtract Money
GO MATH: Ch.3 L11

Vocabulary Keywords: Learning Outcomes:


-Create a table to solve money
Currency problems.
Money -Compute the sum or difference of
money.

What are the different currencies you are familiar with?


Here in UAE, there are lots of tourists coming from
different countries. When they arrive here, they are
converting their money to Dirhams in order for them to
spend and shop easily. In this lesson, we are going to
solve problems involving money with emphasis on adding and
subtracting them.

Rule: The strategy “make a table” helps us to record balances and


deposits and subtract checks.

Example 1:At the end of May, Mrs. Emaan had an account balance of
1,442.37AED. Since then, she has written a check for 163.92AED
and made a deposit of 1,350.00AED. Mrs. Emaan says she has
1,628.45AED in her account. Make a table to determine if Mrs.
Emaan is correct.

Step 1:Read the Problem

Step 2: Identify what is needed in the problem.


We have to determine if Mrs. Emaan’s checkbook balance is
correct.

Step 3: Identify what information you need.


We need that initial balance of her account, amount of check and
amount deposited
82
Step 4: Determine how you will use the information.
We have to subtract the amount of check from the initial account
balance then add the deposit.
Step 5: Solve the problem.

Mrs. Emaan Checkbook


May
1,442.37
Balance
Check 163.92 −163.92
1,278.45
Deposit 1,350.00 +1,350.00
Balance 2,628.45

1,442.37
− 163.92
1,278.45
+1,350.00
2,628.45

Mrs. Emaan’s correct balance is 2,628.45 AED

Example 2:Ghanem is buying for himself and 5 friends. Each bottle


of juice cost 2.25AED. How much does 6 bottles of juice cost? Make
a table to find the cost of 6 bottles of juice.

Step 1:Read the Problem

Step 2: Identify what is needed in the problem.


We need to determine how much 6 bottles of juice will cost.

Step 3: Identify what information you need.


We need to use the price of each bottle and the number of bottles
being bought.

Step 4: Determine how you will use the information.


We have to make a table and then add the amount per bottle until
I find the cost of 6 bottles of juice.

83
Step 5: Solve the problem.

Bottles of
Total
Juice
1 2.25
2 4.50
3 6.75
4 9
5 11.25
6 13.50

Therefore, the total cost of 6 bottles of juice is 13.50 AED.

Exercises 1 – 5:

1) Mr. Rabi teaches second grade. He gets 184.00AED every year to


spend on school supplies. So far, he has only used 113.00AED of it.
How much money is left?

2) The regular price of a plain blanket is 114.63AED. My cousin has a


coupon for 112.57AED off. How much will she pay for the blanket?

3) Raja bought a rug and a coffee table for the living room. The rug
cost 162.88AED, and the coffee table cost 199.70AED. How much
money did Raja spend in all?

4) A teacher wants to buy a new board game for her class. The
regular price is 146.05AED, but today it is on sale for 127.72 off of
the regular price. What is the sale price of the board game?

5) Rani has a balance of 11,800.50AED in his personal checking


account. He pays two bills out of this account: a 150.23AED
electric bill, and a 170.80AED cell phone bill. How much money is
left in Rani’s checking account after he pays these bills?

84
Chapter 3 Lesson 12: Choose a Method

GO MATH: Ch.3 L12

Vocabulary Keywords: Learning Outcomes:


Sum -Select a method to find a decimal
Commutative Property sum or difference.
Difference -Solve word problems involving
Associative Property addition and subtraction of
decimals.

There is more than one way to solve a real life problem. In the problems
involving money, we can use addition to determine that total deposit
that you make then subtract it to the total amount of withdrawals you
have. Or you can add and subtract depending on the flow of accounts in
the balance sheet. In this lesson, we will learn how to choose the
appropriate method to solve problems involving addition and
subtraction of decimals.

Rule: We can choose to use mental computation, properties of


operations, paper and pencil or calculator to find the sum and difference
of decimals.

Example 1:At a track meet, Rayan entered the long jump. His jumps
were 3.25 meters, 2.81 meters, and 2.75 meters. What was the total
distance Steven jumped?

To find the sum of decimals, you can use properties and mental math or
you can use paper and pencil.

Method 1:Use properties and mental math.

Add. 3.25 +2.81 +2.75


3.25 + 2.81 + 2.75
= 3.25 + 2.81 + 2.75 Commutative Property
= (3.25 + 2.75) + 2.81 Associative Property
= 6.00 + 2.81
= 8.81

85
Method 2: Use place-value.
Add:3.25 +2.81 +2.75
1 1
3.25
2.81
+ 2.75
8.81

Example 2:In2011, Abdul Aziz won a gold medal with a long jump of
5.44 meters. In 2012, Nasser Jaffar won the bronze medal with the
jump of 4.31 meters. How much longer was Abdul Aziz’s jump than
Nasser’s?

A. Use place-value B. Use a calculator

5.44
-4.31
1.13

So, Abdul Aziz’s jump was 1.13meters longer than Nasser.

Exercises 1 – 8:

A- Find the sum or difference using any method.

1) 207.51- 196.73 2) 432.93- 309.31

3) 470.79+387.32 4) 147.39+762.85

5) 846.83+728.88 6) 863.39- 540.59

B- Solve.

7) After buying some erasers for 84.49AED as supplies for her shop,
Badia has $28.76 left. How much money did Badia have to begin
with?

8) Basma mixed 1.7 grams of salt into a pot of soup he was cooking.
Before he served the soup, Basma added 0.9 grams of salt. How
much salt did Basma put into the soup in all?

86
C
HAPTER
Chapter 4: Multiply Decimals
4

87
Chapter 4 Lesson 1: Multiplication Patterns
with Decimals
GO MATH: Ch.4 L1

Vocabulary Keywords: Learning Outcomes:


Multiplication Pattern -Recognize a pattern in products
Factor when multiplying by powers of 10.
Product -Solve word problems involving
patterns in multiplication by powers
of ten

Mr. Taher posted this number pattern on the board.

Notice that every time you multiply by another power of 10, we simply
add 0 to the product.

Rule: Use a pattern to find the number of places you need to move the
decimal point in a product. In each increasing power of 10, move the
decimal point one place to the right. In every decreasing power of 10,
move the decimal point one pace to the left.

Example 1:Farah is combining equal-sized rectangles from different


paper patterns for her Art project. Each rectangle has an area of
0.85 of a square inch. If she uses, 1,000 rectangles, what will be the
area of the project?

Use the patterns to find the product.

The project will have an area of 850 square inches.

88
Example 2:Isam is making a scale model of the Burj Al Arab for a
theater set. The height of the building is 321 meters. If the model is
1
of the actual size of the building, how tall is the model?
100
1 x 321 = 321
0.1 x 321 = 32.1
0.01 x 321 = 3.21
Therefore Isam’s model of the Burj AL Arab is 3.21 meters feet tall.

Example 3:Three friends are selling items at arts fair. Hafa makes
145.75AED selling jewelry. Fahad makes 100 times as much as Hafa
makes by selling his custom furniture. Faisal makes a tenth of the
money Fahad makes by selling paintings. How much money does
each friend make?
Hafa:145.75AED
Fahad: 100 ×145.75 Faisal: 0.1×14, 575
1×145.75 = 145.75 1×14,575 = 14,575
10×145.75 = 1, 457.50 0.1×14,575 = 1, 457.50
100×145.75 = 14,575.00

So, Hafa makes 145.75AED, Fahad makes 14,575AED, and Faisal makes
1,457.50AED.

Exercises 1- 3:

A- Complete the pattern.

1) 1 x 2.89 = _____ 2) 1 x 40.36 = _____


10 x 2.89 = _____ 0.1 x 40.36 = _____
100 x 2.89 = _____ 0.001 x 40.36 = _____
1000 x 2.89 = _____

B- 3) Umar is making the cords for the name tags to be used for the
general assembly of all their staffs. If the length of each cord
should be 2.65 feet long, how much cord will be needed if she will
make 1000 pieces?

89
Chapter 4 Lesson 2: Multiply Decimals and
Whole Numbers
GO MATH: Ch.4 L2

Vocabulary Keywords: Learning Outcomes:


Multiply -Model multiplication of whole
Factor numbers and decimals.
Product -Solve word problems involving
decimal multiplication.

The red kangaroo, the world’s largest marsupial, uses its tail for balance
when jumping. Its tail is about 0.52 times as long as its body. Its body is
about 2 meters long. How long is its tail?
Make a model to show how to multiply 2 by 0.52.

What is 2 × 0.52?
Step 1 Step 2
Use hundredths models. Count the number of shaded
Shade 0.52, or 52 hundredths. hundredths. There are 104
Use a different color each time. shaded hundredths. This is
1 whole and 4 hundredths

So, 2 × 52 = 1.04

Example 1:Find the product.


1) 5 × 0.76 2) 0.44 × 3 3) 0.24 × 6

0.76 0.44 0.24


× 5 × 3 × 6
3.80 1.32 1.44

4) 0.16 × 7

0.16
× 7
1.12

90
Exercises 1-15:

A- Find the product using models.


1) 0.673 × 2 7) 0.127 × 5

2) 0.27 × 4 8) 0.114 × 5

3) 0.381 × 3 9) 3.76 × 5

4) 0.571 × 2 10) 22.05 × 5

5) 0.529 × 5 11) 27.412 × 20

6) 0.46 × 6 12) 6.007 × 19

B- Solve.
13) Layla is using a stair-climbing machine set to 4.8 flights of
stairs per minute. How many flights will Layla climb in 6
minutes?
14) Raya walks 0.8 miles on each trip to the park. How far will
Raya walk if she makes 4 trips to the park?
15) Factory XYZ makes 9.5 kilograms of pumpkin pie filling per
minute. How many kilograms of pie filling will the factory make
in 5 minutes?

91
Chapter 4 Lesson 3: Multiplication with
Decimals and Whole Numbers
GO MATH: Ch.4 L3

Vocabulary Keywords: Learning Outcomes:


Multiplication -Draw a model to multiply a decimal
Factor and a whole number.
Product -Use the place value to multiply a
decimal and a whole number.

Suppose a factory released a newly designed mini toy with a mass of 2.5
grams. If there are 6 mini toys made, what is the total mass of the toys?
To solve this problem, we can use the following rule.

Rule: In multiplying decimals by a whole number, draw a quick picture


using partial products, renaming groups as necessary. Use place value to
convert a decimal multiplication to a whole number multiplication and
ensure that the decimal point is properly located in the product.

Example 1:Multiply: 6 × 2.5

Step 1: Estimate the product by rounding the decimal to the nearest


whole number.
6×3 =
18

Step 2: Multiply the tenths by 6.


2.5
×6
3 ← 6 ×5 tenths = 30 tenths,
or 3 ones

Step 3: Multiply the ones by 6.


2.5
×6
2.5
12 ← 6 ×2 ones = 12 ones, or 1 ten and 2 ones

92
Step 4: Add the partial products.
2.5
× 6
3
+12
15

So, 6 mini toys have a mass of 15grams.

Example 2: Multiply: 9 × 1.35

Step 1: Write the decimal factor as a whole number.


Think: 1.35 × 100 =135

Step 2: Multiply as with whole numbers.

Step 3: Place the decimal point.


Think: 0.01 of 135 is 1.35. Find 0.01 of 1,080 and record the product.

×0.01
1.35 ×100
 → 135  → 1.35
× 9 × 9 × 9
? ×100
 → 1,215 ×0.01
 → 12.15

Therefore, the product is 12.15


93
Exercises 1 – 12:

A- Find the product using models and place value.

1) 4 x 6.7 2) 3 x 12.2

3) 7 x 0.12 4) 13 x 1.2

5) 5 x 3.45 6) 4 x 9.28

B- Find the product using place value patterns.

7) 4.87 x 3 8) 3 x 8.91

9) 7.4 x 9 10) 5 x 7.13

11) 12.67 x 5 12) 8 x 34.3

94
Chapter 4 Lesson 4: Multiply Using
Expanded Form
GO MATH: Ch.4 L4

Vocabulary Keywords: Learning Outcomes:


Expanded Form -Draw a model to multiply a decimal
Factor by a whole number in expanded
Product form.
-Use place value to multiply a
decimal and a whole number.

The length of a day is the amount of time it takes a certain planet to


make a complete rotation along its axis. We can solve different problems
involving this using arithmetic. In this lesson, we will use expanded form
of numbers in order to find the product of a whole number and a
decimal.

Rule: Use expanded form to sketch an area model and find the sum of
partial products. Use place value to multiply decimals similar to
multiplying whole numbers then place the decimal point correctly on
the product.

Example 1:There are 9.8 Earth hours for one day on Jupiter. How
many Earth hours are there in 56 days on Jupiter?
Use a model.
Multiply: 56 × 9.8
Think Model Record
Step 1:
Express the factors in
expanded form, and label 9.8
the model. × 56
Step 2: 450 ← 50 × 9
Multiply to find the area 32 ← 50 × 0.8
of each section. The area 54 ← 6×9
of each section 4.8 ← 6 × 0.8
represents a partial 540.8
product.
Step 3:
Add the partial products.
So, there are 540.8Earth hours in 56 days on Jupiter.
95
Example 2:One day on Mercury is equivalent to 58.6 Earth days.
How many Earth days are there in 15 days on Mercury? Use place
value pattern to solve the problem.

Multiply: 15 × 58.6

Step 1: Write the decimal factor as a


whole number. 586
Step 2: Multiply as with whole × 15 58.6
numbers.
2930 × 15
Step 3: Place the decimal point.
1 +5860 879.0
The decimal product is 0.1 or of 8790
10
the whole number product.

So, there are 879Earth days in 15 days on Mercury.

Exercises 1 – 10:

A- Draw a model to find the product.

1) 12 × 0.01 = _______

2) 16 × 0.6 = _______

3) 12 × 0.4 = _______

4) 10.07 × 3 = _______

5) 16 × 0.2 = _______

B- Use place value pattern to find the product.

6) 10.6 × 2 = _______

7) 10.1 × 6 = _______

8) 14 × 0.08 = _______

9) 10.1 × 7 = _______

10) 13.02 × 7 = _______

96
Chapter 4 Lesson 5: Problem Solving:
Multiply Money
GO MATH: Ch.4 L5

Vocabulary Keywords: Learning Outcomes:


Multiply -Draw a diagram to represent
Factor word problems involving money.
Product -Solve money problems.

If the cost of a cola is 1.75 AED and you want to buy a dozen, how much
will you pay in all? One way to solve this problem is by multiplying the
unit price by the number of items. In this lesson, we will apply what we
have learned on using patterns, models and place value in order to
multiply a decimal with a whole number.

Rule: To solve a decimal multiplication problem, use the strategy “draw


a diagram”. Bar models helps you organize the information in the
problem to understand the steps you need to solve it.

Example 1:Tamam spends 13.75AED. Maryam spends 3 times as


much as Tamam. Hadia spends $15.25 more than Maryam. How
much does Hadia spend?

Step 1:Read the Problem

Step 2: Identify what is needed in the problem.


We need to find the amount of money that Hadia spent.

Step 3: Identify what information you need.


We need to the amount spent by Tamam to find the amount spent
by Maryam and Hadia.

Step 4: Determine how you will use the information.


We have to draw a diagram to show the amount Tamam and
Maryam spend to find the amount spent by Hadia

97
Step 5:The amount of money Maryam and Hadia spend depends on the
amount Tamam spends. Draw a diagram to compare the amounts
without solving. Then, use the diagram to determine the amount each
person spends.

Tamam: 13.75
Maryam: 3 × $13.75 = 41.25
Hadia:41.25+ 15.25 =56.50

Therefore, Hadia spent 56.50 Dirhams.

Exercises 1 – 3:

1) Three students in the Biology Club enter a beetroot making


contest. Hani’s beetroot is worth 112.75 AED. Hafza’s beetroot is
worth 5 times as much as Hani’s. Huda’s beetroot is worth 10.25
AED more than Hafza’s. How much is Huda’s beetroot worth?

2) Fahd collects 45.75 AED from his sweets business. If Fawzi


collects 4 times as much as Fahd, how much money would Fawzi
collect?

3) Rabia buys a pair of pants worth 150.25 AED. Her sister Rubi
spends 4.99 AED more for the same pair of pants. Radwa paid the
same price as Rubi but she bought 3 pairs. How much did Radwa
spend?

98
Chapter 4 Lesson 6: Decimal Multiplication

GO MATH: Ch.4 L6

Vocabulary Keywords: Learning Outcomes:


Multiply -Model multiplication of decimals.
Product -Use models to find the product of
two decimals.

In the previous lessons, we have learned on how to multiply a decimal


by a whole number. In this lesson we will learn how to multiply a
decimal by another decimal using model.

Rule: To use a model in multiplying decimals, shade columns of one or


more decimal squares with same colors to represent one factor. Use a
different color to shade rows of the shaded columns to represent the
other factor. Then count or multiply to determine the number of small
squares that are shaded two times.

Example: Multiply 0.2 x 1.4

Step 1: Shade the columns to represent 1.4. Since there are 14 tenths in
1.4, then we have to shade 14 columns.

Step 2: Shade rows that overlap the shaded columns in step 1 to


represent 0.2. Since there 2 tenths in 0.2, then we have to shade 2 rows.

Step 3: Count the number of squares that were shaded two times.
Record the product.

Therefore, 0.2 x 1.4 = 0.28

99
Exercises 1 – 10:

Use the decimal model to find the product.

1) 0.8 × 0.5 = _______

2) 0.8 × 0.7 = _______

3) 0.2 × 0.8 = _______

4) 0.8 × 0.7 = _______

5) 0.8 × 0.2 = _______

6) 0.9 × 0.3 = _______

7) 0.6 × 0.3 = _______

8) 0.4 × 0.5 = _______

9) 0.7 × 0.6 = _______

10) 0.5 × 0.7 = _______

100
Chapter 4 Lesson 7: Multiply Decimals

GO MATH: Ch.4 L7

Vocabulary Keywords: Learning Outcomes:


Multiply -Write a decimal point in
Decimal Point decimal multiplication.
Product -Compute the product of
decimals using place value.

Using decimals in multiplication is very important in solving situations


involving currencies, weights and other measures. In this lesson, we will
learn how to multiply decimals by another decimal and determine the
strategy that we can use to place a decimal point in the product.

Rule: To place a decimal point in a product, multiply the factors the


same way as with multiplying whole numbers and then move the
decimal point one place to the left in the product for each decimal place.
Another way is to use estimation in order to predict what the whole
number should be and use that number to place the decimal.

Example 1:Multiply:1.5 × 2.6

Use place value.


Step 1: Multiply like
whole numbers

Step 2: Place the


decimal point. 26 →×.01 → 2.6 (1 place value)
Tenths are being × 15 →×0.1 →×1.5(1 place value)
multiplied by tenths.
130 3.90(1+1, or 2 place value)
Use the pattern
0.1 × 0.1 . + 260
390 →×0.01
Place the decimal
point so the value of
the decimal is
hundredths.

Therefore the product is 3.9.

101
Example 2:Multiply: 6.8 × 3.12 using estimation.

Step 1: Estimate by rounding each


factor to the nearest whole number.

312
× 68 3.12
2496 × 6.8
+18720 21.216
Step 2: Multiply like whole 21216
numbers.

Step 3: Use the estimated value to


place the decimal point.

The product should be close to your


estimate.

21.216
6.8 × 3.12 =

Exercises 1 – 10:

A- Place the decimal point in the product using the place value.

1) 2.34 × 9.3 2) 0.45 × 2.9

3) 1.75 × 4.6 4) 2.22 × 9.4

5) 7.97 × 6.6

B- Place the decimal point in the product using estimation.

6) 6.35 × 1.1 7) 4.67 × 7.8

8) 8.13 × 3.6 9) 3.87 × 2.1

10) 5.92 × 4.1

102
Chapter 4 Lesson 8: Zeros in the Product

GO MATH: Ch.4 L8

Vocabulary Learning Outcomes:


Keywords: -Write zeros in the product when the
Multiply product has insufficient digits to place the
Zeros in Product decimal point.
Product -Calculate the product of decimals with
zeros in the product

When decimals are being multiplied, the product may have insufficient
digits to place the decimals point. In these situations, write additional
zeros.

Rule: To know if you have the correct number of decimal places in the
product, count the number of decimal place values in each factor and
then add them to determine the correct number of place values the
product should have.

Example 1:Multiply: 0.7 × 0.2

Step 1: Multiply like whole numbers.

Step 2: Determine the placement of


the decimal point in the product.

Since you are getting the product of


tenths by another tenths, the product
will show hundredths.

Step 3:Place the decimal point.


Since there is no enough digits in the
product to place the decimal point,
write zeros to the left of the whole
number product to place the decimal
point.

Therefore, the product is 0.14.


103
Example 2:Multiply: 0.8 × 0.30

Step 1: Multiply like whole numbers.

The factors are 30 hundredths and 8


tenths.

Multiply30 × 8

Step 2: Determine the placement of the


decimal point in the product.

Since you are getting the product of


hundredths and tenths, the product will
show thousandths.

Step 3: Place the decimal point. Write


zeros to the left of the whole number
product.

So, 0.8 × 0.30 is 0.24.

Exercises 1 – 10:

Find the product.

1) 0.06 x 0.8 2) 0.3 x 0.4

3) 0.09 x 0.6 4) 0.05 x 0.4

5) 0.3 x 0.1 6) 0.7 x 0.7

7) 0.06 x 0.9 8) 0.04 x 0.3

9) 0.08 x 0.04 10) 0.07 x 0.09

104
C
HAPTER
Chapter 5: Divide Decimals
5

105
Chapter 5 Lesson 1: Division Patterns with
Decimals
GO MATH: Ch.5 L1

Vocabulary Keywords: Learning Outcomes:


Divide -Recognize patterns in quotients
Pattern when dividing by powers of 10.
Quotient -Use exponents to divide decimals
by powers of ten.

Recall that we used the pattern in powers of 10 in multiplying decimals.


In this lesson, we will learn how to use patterns in dividing a decimal by
a power of 10.

Rule 1: In using a pattern of powers of 10, the decimal point moves one
place to the left every time the divisor increases by a power of 10.

Rule 2: Dividing a power of 10 is similar to multiplying by 0.1, 0.01, and


0.001.

Example 1:Use place value patterns to divide 420 ÷ 1000.

Divide Number of Zeros Move Decimal Point:

420 ÷ 1 = 420 0 0 place to the left

420 ÷ 10 = 42.0 1 1 place to the left

420 ÷ 100 = 4.20 2 2 places to the left

420 ÷ 1000 = 0.420 3 3 places to the left

Therefore, 420 ÷ 1000 = 0.420 or 0.42

We can also use exponents to look for a pattern in dividing decimals.

Recall that 10o = 1 and 101 = 10

106
Example 2:Use exponents to divide 36.4 ÷ 102

Divide Number of Zeros Move Decimal Point:

36.4 ÷ 10o = 36.4 0 0 place to the left

36.4 ÷ 101 = 3.64 1 1 place to the left

36.4 ÷ 102 = 0.364 2 2 places to the left

Therefore, 36.4 ÷ 102 = 0.364.

Exercises 1 – 5:

A- Complete the pattern.

1) 234 ÷ 1 = ______ 2) 46.89 ÷ 1 = _____

234 ÷ 10 = ______ 46.89 ÷ 10 = _____

234 ÷ 100 = _____ 46.89 ÷ 100 = _____

234 ÷ 1000 = _____ 46.89 ÷ 1000 = _____

3) 52.27 ÷ 10o = _____ 4) 28.02 ÷ 10o = _____

52.27 ÷ 101 = _____ 28.02 ÷ 101 = _____

52.27 ÷ 102 = _____ 28.02 ÷ 102 = _____

B- Solve.

5) Rahma used 24.5 pounds of beef in his recipe for today. If she used
one tenth as many pounds of chicken as pounds of beef, how many
pounds of chicken did she use?

107
Chapter 5 Lesson 2: Divide Decimals by
Whole Numbers
GO MATH: Ch.5 L2

Vocabulary Learning Outcomes:


Keywords: -Model division of decimals by a 1 –
Divide digit whole number using base – ten
Pattern blocks.
Quotient -Compute the quotient of a decimal
by a whole number using base ten
blocks.

In lesson 1, we learned how to divide decimals by powers of 10. In this


lesson, we will divide a decimal by a whole number using base – ten
blocks.

Rule: To divide a decimal by a whole number, use the base – ten blocks
or other decimal models to show the dividend. Then share the blocks
equally among the number of groups that is described in the divisor.
Regroup if needed to ensure that the share is made equally. The number
of each group serves a s the quotient.

Example 1:Divide 2.1 by 3 using base ten blocks.

Step 1: Draw a model to represent the dividend 2.1.

Step 2: Draw 3 circles to represent the divisor 3.

108
Step 3: Since we cannot divide 1 by 3, then regroup 1 as tenths. Since
there are 21 tenths in 2.1, we have,

Step 4: Share the tenths equally among the 3 groups. Each group
contain 0 ones and 7 tenths

Therefore, 2.1 ÷ 3 = 0.7

Exercises 1 – 10:

Divide using base – ten models.

1) 2.4 ÷ 3

2) 3.6 ÷ 3

3) 4.4 ÷ 4

4) 6.5 ÷ 5

5) 7.2 ÷ 3

6) 1.4 ÷ 7

7) 8.1 ÷ 9

8) 3.5 ÷ 5

9) 5.6 ÷ 8

10) 6.3 ÷ 7

109
Chapter 5 Lesson 3: Estimate Quotients

GO MATH: Ch.5 L3

Vocabulary Keywords: Learning Outcomes:


Divide -Use compatible numbers greater
Estimate than the dividend to estimate
Quotient quotients in decimal division.
Compatible Numbers - Use compatible numbers greater
than the dividend to estimate
quotients in decimal division.

The length of an adult small intestine is about 6.7 meters when


stretched out. A standard sized front door is about 2 meters high. About
how many doors long is an adult’s small intestine? Sometimes, we do
not need to determine an exact answer because making estimation
would be enough. In this lesson we are going to estimate quotients of
decimals using compatible numbers.

Rule: Use compatible numbers to estimate quotients. In selecting the


appropriate compatible numbers, check the whole number part of the
decimal dividend. If that whole number is less than the divisor, rename
the decimal as tenths or hundredths first then find compatible numbers.

Example: Thirty two students of Mr. Ahmad planned to go to the


theater house for their music and arts project. The total cost for the
tickets is 146.99 AED. About how much money will each student
need to pay for a ticket at the theater?

Estimate: 146.99 ÷ 32

A. Use a whole number greater than the dividend.

Let us use 30 as the divisor. Then determine a number close to and


greater than 146.99 that divides easily by 30.

110
Therefore, each student will pay about 5AED for a ticket.

B. Use a whole number less than the dividend.

Let us use 30 as the divisor. Then determine a number close to and less
than 146.99 that divides easily by 30.

So, each student will pay about 4 for a ticket.

Exercises 1 – 10:

Estimate the quotients of the following:


1) 12.5 ÷ 4

2) 23.7 ÷ 7

3) 3.1 ÷ 9

4) 63.4 ÷ 9

5) 7.84 ÷ 2

6) 124.4 ÷ 6

7) 67.98 ÷ 3

8) 132.67 ÷ 5

9) 8.34 ÷ 2

10) 243.55 ÷ 8

111
Chapter 5 Lesson 4: Division of Decimals by
Whole Numbers
GO MATH: Ch.5 L4

Vocabulary Keywords: Learning Outcomes:


Divide -Calculate the quotient of decimals by
Estimate whole numbers.
Quotient -Solve word problems involving
division of a decimal by a whole
number.

Dividing decimals by whole numbers is very helpful especially when we


are dealing with money. For instance, prices of cookies of same brand
vary according to its weight. By dividing decimals by the number of
grams, we can determine which of the packages is the best buy because
the best buy cost the least amount of money per gram.

Rule: To divide decimal by a whole number, estimate the quotient first.


After estimation, divide like whole numbers. Lastly, use the estimate to
place the decimal point in the quotient.

Example: Use an estimate to divide: 18.72 ÷ 24

Estimate the quotient. 20÷20 = 1


• Divide the tenths.
• Divide the hundredths. When the remainder is zero and there are
no more digits in the dividend, the division is complete.
• Use your estimate to know the location of the decimal point. Place a
zero to show there are no ones.

0.78
24 18.72
- 168
192
- 192
0
Therefore, 18.72 ÷ 24 is 0.78.
112
Exercises 1 – 12:

A- Estimate then divide.

1) 1.29 ÷ 6

2) 9.51 ÷ 6

3) 1.26 ÷ 5

4) 7.79 ÷ 5

5) 9.70 ÷ 2

6) 3.76 ÷ 5

7) 7.77 ÷ 4

8) 2.12 ÷ 2

9) 7.66 ÷ 2

10) 5.09 ÷ 8

B- Solve.

11) You went to the hardware store and bought 7 yellow ropes. The
total length of the ropes was 517.37 meters. How long was each
rope?

12) A candy factory made 419.76 pounds of gummy bears in 4 days.


How much gummy bears, on average, did the factory make per
day?

113
Chapter 5 Lesson 5: Decimal Division

GO MATH: Ch.5 L5

Vocabulary Keywords: Learning Outcomes:


Divide -Model decimal division.
Divisor -Use models to divide a decimal by
Quotient another decimal.

Suppose you need to cut a piece of wire whose length is 3.45 meters into
small pieces whose lengths are equal to 1.15 meters. How many small
wires are there? In the previous lesson, we learn how to divide a
decimal by a whole number. In this lesson, we will learn the division of a
decimal by another decimal using a model.

Rule: To divide a decimal by another decimal using a model, shade


decimal models to represent the dividend. Then cut the model apart into
groups whose sizes are equal to the divisor. The number of same-sized
groups is the quotient of the two decimals.

Example 1:Divide 1.2 ÷ 0.3

Step 1: Shade 12 tenths to represent the dividend 1.2.

Step 2:Divide the 12 tenths into groups of 3 tenths to represent the


divisor 0.3

Step 3: Count the groups. There are 4 groups of 0.3 in 1.2.

Therefore 1.2 ÷ 0.3 = 4

114
Exercises 1 – 10:

Use models to divide the following.

1) 9.6 ÷ 0.8 = _____

2) 3.6 ÷ 0.4 = _____

3) 3.1 ÷ 0.1 = _____

4) 9.5 ÷ 0.5 = _____

5) 6.6 ÷ 0.6 = _____

6) 1.71 ÷ 0.09 = ______

7) 1.76 ÷ 0.02 = ______

8) 0.02 ÷ 0.02 = ______

9) 1.89 ÷ 0.07 = ______

10) 1.26 ÷ 0.06 = ______

115
Chapter 5 Lesson 6: Divide Decimals

GO MATH: Ch.5 L6

Vocabulary Keywords: Learning Outcomes:


Divide -Write the decimal point in the
Divisor correct place in decimal division.
Quotient -Calculate the quotient of a
decimal by another decimal.

Marwan has 2.25 AED. He wants to buy candies that cost 0.25 each. How
many can he but?

Rule: Multiply the dividend and the divisor by the same power of 10 in
order to make the divisor a whole number. The quotient remains the
same as long as you multiply by the dividend and the divisor by the
same power of 10.

So,
2.25 ÷ 0.25 = 9

225 ÷ 25 = 9
Therefore, Marwan can buy 9candies.

Example:Tara hikes on the mountainous region in Ras Al Khaimah.


She plans to hike 1.12 miles. If she hikes at an average speed of 1.4
miles per hour, how long will she hike?

Divide: 1.12 ÷ 1.4


Estimate:1
STEP 1 STEP 2 STEP 3
Multiply the divisor by a Write the decimal point Divide:
10 to make it a whole in the quotient above 0.8
number. the decimal point in the 14 11.2
Then, multiply the new dividend. - 0
dividend by 10 as well. .
112
1.4 × 10= 14 14 11.2
- 112
1.12 × 10 = 11.2
0
So, Tara will hike 0.8 hours.

116
Exercises 1 – 10:

Divide.

1) 3.4 ÷ 0.1 = _____

2) 8.7 ÷ 0.1 = _____

3) 4.8 ÷ 0.2 = _____

4) 0.9 ÷ 0.3 = _____

5) 2.8 ÷ 0.7 = _____

6) 0.86 ÷ 0.02 = ______

7) 1.02 ÷ 0.02 = ______

8) 0.60 ÷ 0.02 = ______

9) 0.63 ÷ 0.09 = ______

10) 0.98 ÷ 0.02 = ______

117
Chapter 5 Lesson 7: Write Zeros in the
Dividend
GO MATH: Ch.5 L7

Vocabulary Keywords: Learning Outcomes:


Divide -Write zeros in the dividend when
Divisor there is insufficient number of
Quotient digits in the dividend.
-Compute the quotient of decimals.

In the final item in a math test of Mr. Rashid, Daania got a final answer of
2.350 while Ali got 2.35. Ali is insisting to Mr. Rashid that Daania’s
answer is wrong and his answer is the correct one. Do you agree with
Ali?

2.350 and 2.35 are equivalent decimals. This means that these two
decimals have the same value. Sometimes, equivalent decimals are used
especially in dividing decimals in cases where the dividend has no
enough digits to complete the division process.

Rule: Write a zero in the dividend when there is insufficient number of


digits in the dividend to complete the division.

Example: Divide 6.2 by 4

Step 1: Divide until you get a number that is less 1.55


than the divisor. 4 6.20
4
Step 2: Place the decimal point and a zero at the 22
end of the dividend. 20
20
Step 3: Place the decimal point in the quotient
20
directly above the decimal point of the dividend.
0
Step 4: Continue dividing.

So, 6.2 ÷ 4 = 1.55

118
Exercises 1 – 10:

Write zeros in the dividend, if needed, and then divide.

1) 2.5÷ 4

2) 3.9 ÷ 5

3) 8.54 ÷ 4

4) 8.1 ÷ 6

5) 3.2 ÷ 5

6) 4.1 ÷ 2

7) 54.35 ÷ 2

8) 156.2 ÷ 11

9) 31.95 ÷ 9

10) 12.46 ÷ 4

119
Chapter 5 Lesson 8: Problem Solving:
Decimal Operations
GO MATH: Ch.5 L8

Vocabulary Keywords: Learning Outcomes:


Divide -Solve multistep decimal problems
Divisor by working backwards.
Quotient -Draw a flowchart to model the
situation and solution in the
problem.

Let us apply all the knowledge we have learned in performing


operations on decimals in solving multi-step word problems.

Rule: Use the strategy “work backwards” using a flowchart to determine


the unknown value in the problem.

Example: Last week, Nasreen spent a total of 20.00 in a school fair


where all items are discounted. She spent 9.85AED for toys,
5.75AED for snacks, and the rest for 2 pens that were on sale at the
school fair. If each pen had the same cost, how much did each pen
cost?

Step 1:Read the Problem

Step 2: Identify what is needed in the problem.


We need to find the cost of each pen.

Step 3: Identify what information you need.


We need the total amount that Nasreen spent worth 20.00AED,
the toy the costs 9.85 AED, and the snacks that costs 5.75AED. The
rest are for2 pens.

Step 4: Determine how you will use the information.


Work backward from the amount Nasreen spent to find the cost of
each pen.

120
Step 5: Solve the problem.

4.40 AED ÷ 2 = 2.20 AED


Therefore, the cost of each pen is worth 2.20 AED

Exercises 1 – 2:

1) Obaidiyah spent 55.32 AED for school supplies. She spent 1.75
AED for eraser, 12.96 AED for markers, and 3.23 AED for a black
pen. She also purchased 2 notebooks. If each notebook have the
same selling price, what is the cost of each notebook?

2) Qadi’s initial account balance in the bank is 35.89 AED. He


received a salary (with some deductions) worth 4456.38AED.
Then he withdraws cash worth 1200 AED for his allowance and
2000AED for the house rent and spent 526.92 in groceries. If he
will give the remaining balance of his account to his 2 children
equally, how much will each child receive?

121
122
C
HAPTER
Chapter 6: Add and Subtract
6
Fractions with Unlike Denominators

123
Chapter 6 Lesson 1: Addition with Unlike
Denominators
GO MATH: Ch.6 L1

Vocabulary Keywords: Learning Outcomes:


Fraction -Recognize fractions with different
Addition denominators.
Unlike Denominators -Use models to add fractions with
different denominators.

1
If we want this pizza into 8 equal parts, then each part is
8
of the whole pizza. Also, the sum of all the fractional parts is
shown below,
1 1 1 1 1 1 1 1 1+1+1+1+1+1+1+1 8
+ + + + + + + = = = 1.
8 8 8 8 8 8 8 8 8 8
But what if we want to combine pizzas with different
partitions? How are we going to add them?

Rule: To add fractions with different denominators using models, trade


fraction strips of fractions with different denominators for equivalent
strips of fractions with like denominators.

Example: Use fraction strips to determine the sum of 3/10 and ½.

Step 1: Use a ½ strip and three pieces of 1/10 fraction strips to model
fractions with different denominators.

Step 2: Trade the ½ strip for five pieces of 1/10 strips.

3 1 3 5
+ = +
10 2 10 10
Step 3: Add the fraction strips with the same denominators.

3 5 8
+ =
10 10 10
Step 4:Express the answer in simplest form.

8 4
=
10 5
124
1
1 1 1 1
2 10 10 10
1 1 1 1 1 1 1 1
10 10 10 10 10 10 10 10
SUM

3 1 4
Therefore, + =
10 2 5

Exercises 1 – 5:Use fraction strips to find the sum.

3 2
1) +
5 15

5 1
2) +
6 3

1 5
3) +
8 16

7 9
4) +
8 12

3 1
5) +
7 3

125
Chapter 6 Lesson 2: Subtraction with Unlike
Denominators
GO MATH: Ch.6 L2

Vocabulary Keywords: Learning Outcomes:


Fraction -Use models to subtract fractions
Subtraction with different denominators.
Unlike Denominators -Express the difference in simplest
form.

To subtract fractions with same denominator, we simply subtract the


numerators then copy the same denominator. However, if the
denominators are not the same, the procedure is different. In this lesson,
we will learn how to subtract fractions with different denominators
using models.

Rule: Use models like fraction strips to model subtraction of fractions


with different denominators. To subtract fractions with different
denominators using models, trade fraction strips of fractions with
different denominators for equivalent strips of fractions with like
denominators.

Example: Use fraction strips to determine the difference of7/10


and1/5.

Step 1: Use the 7/10 fraction strips to model the first fraction.

Step 2: Trade one 1/5 strip into two 1/10 strips.

7 1 7 2
− = −
10 5 10 10
Step 3: Subtract by taking away 2/10

7 2 5
− =
10 10 10
5 1
In simplest form, =
10 2

126
1
1 1 1 1 1 1 1
10 10 10 10 10 10 10
?
1
5

7 1 1
Therefore, − =
10 5 2

Exercises 1 – 6:

A- Use fraction strips to subtract the following fractions. Express


your answer in simplest form.

3 1
1) −
5 10
3 1
2) −
4 2
7 1
3) −
12 2
1 1
4) −
2 5
2 1
5) −
3 9
B- Solve.

6) Dana placed an order for 7/10 of a sack of brown lentils and 1/2
of a sack of green lentils. How much more brown lentils did Dana
order?

127
Chapter 6 Lesson 3: Estimate Fraction Sums
and Differences
GO MATH: Ch.6 L3

Vocabulary Keywords: Learning Outcomes:


Sum -Estimate fraction sums and
Benchmark differences using a number line.
Difference -Estimate fraction sums and
Estimate differences using mental math.

7
The length of an adult small intestine is approximately 6 meters long.
10
1
If the length of the large intestine is 1 meters long, what is the
2
estimated difference in their lengths? We can use the following rule to
make reasonable estimates in the sum or difference of fractions.

Rule: To make reasonable estimate in the sum or difference of fractions,


round the fractions using benchmarks like 0, ½ or 1 on a number line, or
compare the numerator to the denominator and round the fraction to its
benchmarks. Then find the sum or difference of the rounded numbers.

1 7
Example 1:Estimate the sum of + using the number line.
6 8
1 1 1
Step 1: Place a point at on the number line. is between 0 and . S
6 6 2
Since the fraction is closer to 0, then round the fraction to 0.

128
7 1
Step 2: Place a point at on the number line. The fraction is between
8 2
7
and 1. Since the fraction is closer to the benchmark 1, then round the
8
fraction to 1.

Step 3: Find the sum of the rounded fractions.


1
→0
6
7
+ →1
8

Therefore, the estimated sum is 1.

8 5
Example 2:Use mental math to subtract −
9 12
8
Step 1:Round . The numerator is about the same as the denominator.
9
8
Round to 1.
9
5
Step 2:Round . The numerator is about half the denominator. Round
12
5 1
to
12 2

Step 3: Subtract the two rounded numbers.


1 1
1- =
2 2
8 5
Therefore, − is about ½.
9 12

129
Exercises 1 – 5:
Estimate the sum or difference using number line and mental math.
2 1
1) +
3 5

5 1
2) −
6 2

4 1
3) −
5 3

5 1
4) −
8 4

7 1
5) −
10 4

130
Chapter 6 Lesson 4: Common Denominators
and Equivalent Fractions
GO MATH: Ch.6 L4

Vocabulary Keywords: Learning Outcomes:


Equivalent Fractions - Write equivalent fractions using
Least Common Denominator the least common denominator.
-Recognize common denominators.

Suppose Aisha planted two gardens. One had three sections of


vegetables and the other had five sections of fruits. She plans to divide
both gardens into more sections so that they have the same number of
equal-sized sections. How many sections will each garden have? To
solve this problem, we can find a common denominator or a common
multiple of two or more denominators to write fractions that name the
same part of a whole.
Rule: To rewrite two fractions with the same denominator, multiply the
two denominators or list the multiples of each denominator and then
find the common multiple. Then rewrite the fractions using the common
denominator.
Definition: The least common denominator is the least common
multiple of two or more denominators.

Example 1: Referring to the problem stated above, multiply the


denominators to determine the number of sections.
Plan Solution
• Multiply the denominators to
find a common denominator.
1
A common denominator of
3
1
and is 15.
5
1 1 1
Divide each into fourths and • Write and as equivalent
3 3 5
1 fractions using the common
divide each into thirds, each of the
5 denominator.
wholes will be divided into the same=1 5 1 3
=
size parts, twelfths. 3 15 5 15

So, both gardens will have 15 sections.

131
1 1
Example 2: Use a list to find the common denominators of and
3 5
• Make a list of the first eight nonzero multiples of 3 and 5.
Multiples of 3: 3, 6, 9, 12, 15 , 18, 21, 24,27, 30
Multiples of 5: 5, 10, 15 , 20, 25, 30
• Box the common multiples.
• Use of the common multiples as a common denominator to write
1 1
equivalent fractions for and .
3 5
1 5 10 1 3 6
= or = or
3 15 30 5 15 30

3 1
Example 3:Find the least common denominator of and .
4 12

Use the least common denominator to write an equivalent fraction for


each fraction.

Step 1:

List nonzero multiples of the denominators. Determine the least


common multiple.
Multiples of 4: 4, 8, 12, 16, 20, 24, 28…
Multiples of 12: 12, 24, 36, 48, 60, 72,..
3 1
So, the least common denominator of and is 12.
4 12
Step 2:

Write an equivalent fraction for each fraction using the least common
denominator,

3 9 1
can be rewritten as and will be the same since the least
4 12 12
common denominator is its denominator itself.

132
Exercises 1 – 7:

A- Use the least common denominator to write an equivalent


fraction for each fraction.

1 4
1) ,
3 5
6 1
2) ,
7 2
1 2
3) ,
6 5
5 1
4) ,
9 3
1 3
5) ,
8 4
5 2
6) ,
7 3
B- Solve.
2 1
7) A recipe calls for cup of white sugar and cup of milk. Find the
3 4
least common denominator of the fractions used in this recipe.

133
Chapter 6 Lesson 5: Add and Subtract
Fractions
GO MATH: Ch.6 L5

Vocabulary Keywords: Learning Outcomes:


LCD -Use a common denominator to add
Sum and subtract fractions with
Difference different denominators.
-Express the sum or difference in
simplest form.

2 1
Suppose you have kilogram of beef meat and your sister has
3 4
kilogram of beef meat, what is the total weight of the beef? To solve this
problem, we have to add the two fractions using the rule below.

Rule: When you add or subtract two fractions with unlike denominators,
you need to make the denominators the same. Find the least common
denominator (LCD), and change the fractions to like fractions with that
denominator.

2 1
Example 1:Add  n
3 4

Step 1

Find the multiples of both denominators to determine LCD.

3: 3, 6, 9, 12, …
4: 4, 8, 12, …
The LCM of 3 and 4 is 12. So, the LCD of

Step 2

Use the LCD to make like fractions. Multiply the numerator and
denominator by the same number

2 = 2× 4 = 8
3 3× 4 12
1 = 1×3 = 3
4 4 ×3 12
134
Step 3
Add the fractions.
8
12
3
+
12
11
12

11
So, n =
12

2 1 11
So, the sum of + =
3 4 12

This answer is in simplest form.

To subtract fractions with unlike denominators, follow these 3steps.


However, in Step 3, subtract the fractions and write the answer in
simplest form.

Example 2:Find the sum or difference.

1 2 2 1
1) + 2) −
3 6 5 6
2 × 6 1× 5
= −
1× 2 2 5× 6 5× 6
= +
3× 2 6 12 − 5
=
2+2 30
=
6
7
=
4 30
=
6
2
=
3

135
4 5 3 1
3) − 4) +
6 8 4 2

4×8 5× 6 3 1× 2
= − = +
6×8 8× 6 4 2× 2
32 − 30 3+ 2
= =
48 4

2 5
= =
48 4
1
=
24

Exercises 1-21:

A- Find the sum or difference. Write your answer in simplest


form.

2 1 3 1 1 2 1 5
− + + +
1) 5 4 2) 4 2 3) 3 6 4) 9 6

2 1 3 1 9 3 3 1
− − − +
5) 7 14 6) 8 4 7) 11 22 8) 5 2

2 5 2 3 6 2 9 3
+ + − −
9) 13 13 10) 11 11 11) 7 5 12) 10 5

6 3 2 1 2 5 9 3
− − 15) − −
13) 7 5 14) 9 7 3 12 16) 10 5

11 3 9 5 2 4 6 2
− − 19) − −
17) 12 4 18) 11 11 3 9 20) 7 3

B- Solve.

21) Mohamed walks 7/8 of a mile to school. Ali walks 1/2 of a mile to
school. How much farther does Mohamed walk than Ali?

136
Chapter 6 Lesson 6: Add and Subtract Mixed
Numbers
GO MATH: Ch.6 L6

Vocabulary Keywords: Learning Outcomes:


Sum -Calculate the sum and difference of
Mixed Numbers mixed numbers with different
Difference denominators.
LCD -Solve word problems involving
addition and subtraction of mixed
numbers.

Firas and Ghassan are going to put up a tent. They need two pieces of
1 1
rope to secure the tent. One has to be 3 feet long and the other 2
2 4
feet long. How much rope do they need?

To find the answer, you must add 3 21 + 2 14 .


You can add mixed numbers by following steps:

Step 1
Add the whole numbers. 3 + 2 = 5

Step 2
Find the LCD. Write equivalent fractions. Add the fractions.

1 12 2
multiples of 4: 4, 8, 12, … = =
2 2 2 4
1 1 1 1
multiples of 2: 2, 4, 6, … = =
4 4 1 4
1 1 2 1 3
+ = + =
2 4 4 4 4

Step 3
Add the sum of the whole numbers to the sum of fractions. Write the
answer in simplest form if needed.
3
5+ = 5 34
4
1
So, 3 21 + 2 = 5 34
4

137
Example 1:Find the sum.

1 1 2 3
1) 2 + 4 2) 4 + 5
2 4 5 10
1 1 2 3
2 +4 4 +5
2 4 5 10

1 1 2 3
( 2 4)
=+ + (4 5
=+ ) +
2 4 5 10

2 +1 2× 2 + 3
=6 =9
4 10

3 4+3
=6 =9
4 10

7
=9
10

1
Example 2:Samia cut out a pattern for a new blouse from the 3
2
yards of material she bought. The pattern used 2 1 yards. How
3
much material is left?

You can answer the question by subtracting, 3 21 - 2 13


To subtract mixed numbers, follow these steps.

Step 1 Step 2

Find the LCD of the fractions Change the fractions into like
by listing the multiples of each fractions with 6 as the denominator.
number.

1×3 3 1×2 2
Multiples of 2: 2, 4, 6, 8, … = =
2×3 6 3×2 6
Multiples of 3: 3, 6, 9, 12, …

Since 6 is the first common multiple, it is the least common multiple.

138
Step 3 Step 4
Subtract the fractions. Subtract the whole numbers.

3 21 = 3 63 3 21 = 3 63

- 2 13 = 2 26 - 2 13 = 2 26

1
6
1 16

So, Samia has 1 16 yards left.

Example 3:Find the difference.

3 1 1 5
1) 5 − 4 2) 7 − 3
4 2 2 18
3 1 1 5
5 −4 7 −3
4 2 2 18

3 1 1 5
(5 − 4)
= −
4 2
( 7 3)
=− −
2 18

3 −1× 2 1× 9 − 5
=1 =4
4 18

3− 2 9−5
=1 =4
4 18

1 4
=1 =4
4 18
2
=4
9

139
Exercises 1-13:
A- Add or subtract the mixed numbers

1 1 1 2 1 1
1) 3 − 2 2) 4 + 1 3) 5 − 2
2 3 3 3 2 4

1 2 1 1 1 2
4) 6 + 2 5) 2 + 3 6) 6 + 2
3 3 4 5 7 5

1 2 2 1 1 1
7) 9 + 2 8) 3 + 6 9) 4 + 3
5 5 3 4 3 4

10) Add 11) Subtract

B- Solve.

12) Hoping to be achieving the title of Employee of the Month, Dana


called the names from 10 5/12 pages of the phone book last week.
This week, she called the people listed on another 11 5/12 pages of
the same phone book. How many pages worth of people did Dana call
in all?
13) In the morning, Youssef drove to the store and used 7 2/5 gallons
of gas. In the afternoon, he drove to the movie theater and used 9 2/5
gallons of gas. How much gas did Youssef use in all?

140
Chapter 6 Lesson 7: Subtraction with
Renaming
GO MATH: Ch.6 L7

Vocabulary Keywords: Learning Outcomes:


Subtraction -Use renaming to find the
Renaming difference of two mixed numbers.
-Solve a word problem involving
subtraction of mixed numbers
with renaming.

Suppose you want to determine the difference between the length of


your turtle shell and the length of an average turtle shell in the pet shop
for an investigatory project in science. If the length of the shell in the pet
shop is 5 ½ feet and your pet has 3 ¾ feet, what is the difference in their
lengths? To solve this, we have to subtract 5 ½ - 3 ¾. One way to do this
is by renaming the mixed numbers before subtracting them.

Rule: To use renaming in finding the difference between two mixed


numbers, first step is to write equivalent fractions using a common
denominator. Second step is to use multiplication and addition to
rename each mixed number as a fraction that is greater than 1. Lastly,
subtract the fractions and express the answer in simplest form.

Note: You can also rename both mixed numbers before you subtract.

1 5
Example 1:Rename the first mixed number then subtract 3 − 1
2 6

Step 1: Estimate the difference.


1 1
3 −2=1
2 2
1 6 18
=3 =
3 2
2 12 12
Step 2: Determine the common denominator. Use
5 10 10
this to write equivalent fractions with the same −1 = −1 = −1
6 12 12
denominator.
8 2
1 =1
12 3
141
6
Step 3: Rename 3 as a mixed number with a fraction greater than 1.
12
6 6 12 6 18
3 =1+1+1+ =1+1+ + =2
12 12 12 12 12
6 18
So, 3 =2
12 12
Step 4: Subtract the fractions then find the difference of the whole
numbers. Write the difference in simplest form and check using your
estimate to see if the answer is reasonable.

Exercises 1 – 11:
A- Estimate and rename the first mixed number or both mixed
numbers then subtract. Write the difference in simplest form.
2 7
1) 6 − 3
13 26
15 1
2) 9 − 2
64 8
9 11
3) 5 − 2
66 22
5 13
4) 7 − 4
49 14
1 13
5) 6 − 3
10 15
20 7
6) 5 − 4
77 11
10 1
7) 4 − 3
78 3
1 2
8) 9 − 2
3 9
3 17
9) 5 − 2
11 55
1 7
10) 7 − 4
43 86

B- Solve.

11) Fatema drew a green line that was 2 5/6 inches long. Then she
drew a white line that was 1 1/6 inches long. How much longer
was the green line than the white line? Rename the fractions first
then subtract. Write your answer in simplest form.

142
Chapter 6 Lesson 8: Patterns with Fractions

GO MATH: Ch.6 L8

Vocabulary Keywords: Learning Outcomes:


Sequence -Describe a pattern or create a
Patterns sequence with fractions.
-Write the next terms in fraction
sequences.

Sarah used 2 ½ cups flour in baking cookies on Monday. Then she used
3 cups of flour on Tuesday and then 3 ½ cups of flour on Wednesday. If
this pattern continues, how many cups of flour will she use on Friday?

Patterns are everywhere, and so with patterns with fractions. You can
use addition and subtraction operations in cases such as this.

Rule: To describe a pattern, write the terms as equivalent fractions with


the same denominator. Then determine the difference between the
consecutive terms and write a rule. Lastly, use the generated rule to
extend the sequence or to find the missing terms. Create a sequence if
the rule and the first term are given.

Example: Find the unknown terms in the sequence.

3 9 3 3 7
2 ,2 ,2 ,2 , ___, ___, ___,
4 16 8 16 16

Step 1: Write the terms in the sequence as equivalent fractions with a


common denominator.
12 9 6 3 7 4
1, , 1 , 1 , 1 , ? , ? , ? , ,
16 16 16 16 16 16

Step 2:Write a rule describing the pattern in the sequence.

Note that we use addition to describe a sequence that increases and


subtraction if the fraction decreases.
3
Rule: subtract
16

143
Step 3: Use your rule to find the unknown terms. Then complete the
sequence above.

3 9 3 3 13 5 7
Therefore, 1 ,1 ,1 ,1 , 1, , ,
4 16 8 16 16 8 16

Exercises 1 – 4:

A- Write a rule for the sequence.

3 19 23
1) , , ,...
4 20 20

5 1 1
2) ,1 ,1 ,...
6 12 3

B- Write the first three terms of the given sequence.

3) Rule: Add 1 ¾

a. 1 7/8

b. 2 6/7

4) Rule: Subtract 2/3

a. 27/12

b. 3 ¼

144
Chapter 6 Lesson 9: Practice Addition and
Subtraction and Properties of
Addition
GO MATH: Ch.6 L9,10

Vocabulary Keywords: Learning Outcomes:


Commutative Property -Recognize the commutative and
Associative Property associative properties of addition.
-Use properties of addition to
solve problems.

Suppose you have an investigatory project about caterpillars and the


three caterpillars that you caught have sizes 1 inch ½ inch and 2/3 of an
inch. What is the sum of their lengths? In solving problems such as this,
we have plan first on what to do before we execute them. There are also
rules and properties that we have to consider. These are as follows.

Commutative Property of Addition

The order of the addends will


not affect the sum

Associative Property of Addition

Grouping the addends will not


affect the sum

145
Example 1:Use the properties of addition to add the following
fractions.
3 7
10 + 4 + 6
5 Given
8 8 8
3 5
10 + 6 + 4
7 Commutative Property
8 8 8

 3 5 7 Associative Property
 10 8 + 6 8  + 4 8
 

8
16 + 4
7 Add
8 8

17 + 4
7 Simplify
8

21
7 Add
8

Example 2:Aaliyah jogs at the park every morning. She wants to


5
complete a total of 7 miles distance. On Monday, she jogs 1 miles,
6
1
On Tuesday, she jogs 2 miles. How many more miles will she need
3
in order to reach her target?
Start with writing an equation to represent the problem.
Let m be the number of miles that she needs to reach her target.
5 1
So the working equation is: m + 1 + 2 = 7
6 3
Then work backwards to determine the value of m.
5 1
7 - 1 -2 = m
6 3

6 1 7
7 → 65 → 4
6 6 6
5 5 1 2
−1 → 1 -2 → −2
6 6 3 6
1 5
5 2
6 6

5
2
Therefore, she has to jog 6 miles more.

146
Exercises 1 – 5:

A- Use the properties of addition to find the following.

 2 1
 2
1)  1 +  + 3
 7 3 3

2 2 3
2)  +  + 3
5 9 5

7 1 1
3) 2 + 4 + 1
8 9 8

B- Solve the following.

4) A school wants to make a new playing area by cleaning up a


vacant lot that is shaped like a rectangle. They give the job of
planning the playground to a group of students. The students
decide to use 1/4 of the playground for a volleyball court and 3/8
of the playground for a soccer field. How much is left for the
swings and play equipment?

5) You give 1/3 of a pan of cake to Sameera and 1/6 of the cake to
Abdul. How much of the cake did you giveaway?

147
148
C
HAPTER 7
Chapter 7: Multiply Fractions

149
Chapter 7 Lesson 1: Fraction Multiplication

GO MATH: Ch.7 L1,2,3,4,5,6

Vocabulary Keywords: Learning Outcomes:


Multiply -Compute the product of fractions
Product using models
Factors -Calculate the product of fractions and
Area Model whole numbers
Fraction -Compute the product of two fractions
-Relate the side of the product and the
size of one factor when multiplying
fractions.

Earth revolves around the sun for 364 ¼ days. If Earth makes 3
complete revolutions around the sun, how many Earth days does it take?
To solve this problem, we have to multiply them. In multiplying
fractions, there are several rules to consider. Take note of the following
rules.

Rule 1: To multiply a fraction by a whole number using models, find first


a fractional part of a group. Place the number of equal-sized fraction
bars indicated by the denominator under the whole, and then check the
number of equal-sized bars indicated by the numerator, then solve. To
determine the groups of fractional part, Use another model for fractions
and shade the fractional part of each group then solve.

Rule 2:To determine the product of a fraction and a whole number


without the use of fraction models, multiply the whole number and the
numerator. Then place the product over the denominator and express
the product in simplest form.

2
Example 1: Hisham has 9 baseball cards. He gave of them to his
3
friend Nader. How many baseball cards did he give to Nader?

2
You can answer the question by multiplying ×9 .
3

150
To multiply a fraction with a whole number you can use a model:

Step 1
Draw 9 rectangles to show
the cards.

Step 2
The denominator of the
fraction 2 is 3. This means
3
there are 3 equal parts, so
divide the rectangles into 3
equal groups.

Step 3
The numerator of the
fraction
2
is 2. This means there
3
are 2 parts given, so shade 2of the groups.

Step 4
Count the shaded rectangles, or cards. There are 6 cards.

2
So, ×9=6
3

Example 2: Find the product.

2 3
1) 3 × 2) 5 ×
5 8

3× 2 5× 3
= =
5 8

6 15
= =
5 8
1 7
=1 =1
5 8
151
1 4
3) 11× 4) 16 ×
9 15

11× 1 16 × 4
=
9 15

11 64 4
= =4
9 15 15
2
=1
9

3
Example 3: Daisy has 12 picture frames. She gave to her friends.
6
How many frames did she give?

3
12 × =6 picture frames
6

3 3
Example 4:Multiply ×
5 4

To multiply fractions you can use a rectangle model.


Follow these guidelines:

•Draw a rectangle and divide


the rectangle into 4 equal columns.
3
This is for the denominator of .
4

152
• Shade 3 of the columns.
3
This is for numerator of .
4

• Divide the rectangle in 5 equal rows.


This is for the denominator of 3 .
5

• Shade 3 of the rows with diagonal lines.


This is for the numerator of 3 .
5

• Count how many pieces the rectangle is


divided into. There are 20 pieces.
This is a new denominator.

• Count how many pieces have overlapping


lines and shading. There are 9.
This is the new numerator.

3 3 9
So, × =
5 4 20

Rule 3: To find the product of two fractions we multiply the numerator


by the numerator and the denominator by the denominator

a c a × c ac
×= =
b d b × d bd
The final product should be in simplest form or a mixed number.

Example 5:Find the product.


1) 1 3
×
1) 2 3
×
6 4
5 7
1× 3 3 1
= =
2×3 6 6 × 4 24 8
=
5 × 7 35

153
2) 4 5
×
7 2
×
3 8 3) 9 3

4 × 5 20 5 7 × 2 14 2
= = = =
3 × 8 24 6 9 × 3 27 3

Let us compare fraction factors and its product.

Rule 4: When you multiply 1 and any fraction, the product is equal to the
fraction. If you multiply a fraction by a fraction that is less than 1, then
the product will be less than either factor. When you multiply a fraction
by another fraction greater than 1, then the product is greater than the
fraction and less than the number greater than 1.

Example 6: Multiply the following using a number line.

A. 1 x ¼

Therefore, 1 x ¼ = ¼ , the same fraction.

B. ¼ x ½

1/8

Therefore, ¼ x ½ = 1/8, which is less than the two fractions.

154
Exercises 1-22:

A- Multiply

6 2 11 2
1) ×3 2) ×5 3) ×4 4) ×3
7 3 12 9

3 5 3 2
5) ×2 6) × 10 7) × 15 8) ×2
7 6 5 5

B- Solve.
9) Multiply each fraction by each number that appears at the top of the
row. Write each answer in the simplest form. Complete the chart
below.

× 4 5 6 8
1
4

3
5

8
12

13
14

C- Find the products

2 14 7 3 7 22
10) × 11) × 12) ×
7 15 9 14 11 49

5 2 3 3 14
13) × 10 14) × 15) ×
6 3 4 7 15

155
D- Circle the two fractions whose product equals the fraction in
the next column and simplify the answer if possible.

11 9 8 3 88
16) , , ,
13 15 9 8 117

1 2 14 12 120
17) 3 , , ,
3 8 15 11 33

4 3 2 6 18
18) , , ,
9 49 50 28 1372

5 6 7 3 42
19) , , ,
12 13 15 11 195

E- Complete the sentence with the expressions equal to, greater


than or less than.

6 6
20) × 1 will be___________
7 7
8 3 3
21) × will be___________
9 4 4
2 1 2
22) × will be___________
9 2 9

156
Chapter 7 Lesson 2: Multiply Mixed
Numbers
GO MATH: Ch.7 L7,8,9,10

Vocabulary Keywords: Learning Outcomes:


Multiply -Use a model to multiply two mixed
Product numbers and find the area of a
Factors rectangle
Mixed Numbers -Relate the size of the product with
its factors greater than 1
-Compute the product of two mixed
numbers
-Solve word problems dealing with
mixed numbers

Suppose have a rectangular piece of cardboard whose dimensions are


12 cm and 4 cm. How are we going to find the area of this rectangle?
Recall that the area is equal to the product of the base and height, or the
product of the length and the width. So, the area of this rectangle is 48
square centimeters. What if the dimensions of the rectangle are
fractions? We can use area models to multiply mixed numbers and
problems involving areas with fractional dimensions.

Rule 1: To use area models in multiplying mixed numbers or fractions,


shade the rectangle with fractional unit tiles and get the product of the
number of tiles and area of each tile.
1 1
Example 1: Multiply 2 ×
4 3

1
Step 1: Model 2
4

157
Step 2:Shade the model twice to represent the 2nd factor 1/3

4 4 1
The fractional parts are , and . Therefore, the product of
12 12 12

1 1 4 4 1
2 × = + +
4 3 12 12 12

9
= (9 is the number of squares that are shaded twice)
12

Rule 2: To multiply mixed numbers without using area models, change


the mixed numbers to fractions greater than 1. Then find the product of
the numerators and the product of the denominators.

1
Example 2: Multiply: 14 × 3 . Write the product in simplest form
6

Step 1: Rewrite the whole number


and mixed number as fractions.
1 14 19
14 × 3= ×
Step 2:Find the product of the two 6 1 6
fractions 266
= = 44
1
6 3
Step 4:Express the product in
simplest form.

1 1
Therefore, 14 × 3 =
44 .
6 3

You can use a number line to compare and relate the product with its
factors.

Rule 3: When you multiply a fraction whose value is greater than 1 by


another fraction that is less than 1, the product will be less that the
factor whose value is greater than 1 and greater than the factor whose
value is less than 1.

158
Example 3:Complete the sentence with the expressions greater
than, less than, of equal to.
3 1 3
× 1 will be _______
4 2 4

This is the product of a fraction that is less than 1 and a fraction that Is
greater than 1. According to rule 3, the product should be greater than
3 1
and less than 1 . So, the answer is greater than.
4 2

Example 4:If the area of a rectangular piece of cardboard for your


project is 300 square inches and the length should 11/3 of the
width, what will be the dimensions of the cardboard?

Length
Make a
1
(1 of the length) Check the area Review
Guess
3
1 1
1 1 10 × 13 =
133 Try a longer
10 1 × 10 =
13 3 3
3 3 width.
very low
1 1
1 1 25 × 19 =
481 Try a shorter
19 1 × 19 =
25 3 3
3 3 width.
very high
15 × 20 =
300
1
15 1 × 15 =
20 equal to the given
3
area

Therefore, the rectangular cardboard should have a length of 20 inches


and a width of 15 inches.

159
Exercises 1 – 11:

A- Multiply the following mixed numbers using area models.


3 1 8 3
1) ×1 2) 2 ×
4 3 9 4

3 4 5 1
3) 5 × 4) 1 ×
8 5 9 5

B- Multiply the following mixed numbers.


3 7 1 1
5) ×4 6) 4 ×
4 12 2 6

7 2 1 3
7) ×8 8) ×6
12 3 10 4

C- Complete the sentence with the expressions greater than, less


than, of equal to.
1 1 1
9) × 6 will be _______ 6
3 4 4

1 4 1
10) × 5 will be ______
5 7 5

D- Solve:

11) If the area of the painting at Ali’s room is 432 square meters and
the width is 1/3 of the length, what are the dimensions of the
painting?

160
C
HAPTER 8
Chapter 8: Divide Fractions

161
Chapter 8 Lesson 1: Interpret Division With
Fractions
GO MATH: Ch.8 L1,2,3,4,5

Vocabulary Learning Outcomes:


Keywords: -Compute the quotient of a whole
number and a fraction and a fraction
Quotient by a whole number.
Dividend -Solve division problems by
Divisor multiplication.
Halves -Interpret a fraction as division and
Fourths solve whole number division
problems that result to a fraction or
mixed number.

Suppose Ahmad cut his pizza into fourths, how many ¼ - sized pizza
pieces would there have been? In this lesson, we will learn different
strategies on how to divide a whole number by a fraction and a fraction
by a whole number.

One way is to use multiplication.

Rule 1: To solve division problem using multiplication, draw a rectangle


to represent each whole unit. Then use the fraction in the problem to
divide each rectangle into equal parts. Lastly, write a multiplication
equation to represent the diagram you built and then solve the problem.

Example 1:Ahmad’s brother Mustafa, has 2 rectangular cakes. He


divides the cakes into halves for his family. How many persons can
receive a ½ - size cake from Mustafa?

Since there are two cakes given, then create 2 rectangles to represent
the 2 cakes. Then cut each rectangle into halves.

162
To determine the number of halves in the 2 rectangles, we have to
multiply the number of halves by the number of rectangles.

2 ÷ ½ = 2 x 2 = 4. Therefore, 4 members of the family will receive a piece


of cake from Mustafa.

A fraction can also be expressed as a division of two numbers.

Rule 2: The numerator of a fraction represents the number of items


being divided, while the denominator represents the number of equal
pieces into which the item is being divided. The fraction and the
quotient of the numerator and the denominator represent the same size
or value of each piece after the division.

Example 2:Mr. Ibrahim brought a 135 – inch piece of wire in their


Physics class. If he cuts the wire equally among his 7 students, how
much wire will each student get?

The situation can be expressed in two ways, as a division and as a


fraction.

By division, we have,
2
135 ÷ 7 = 19 Divide the two numbers. Express the remainder as a
7
fraction

By fraction, we have,
135 2
= 19 Write as a mixed number in simplest form.
7 7

2
19
Therefore, each student will receive 7 inches of wire

163
You can draw a diagram in order to solve problems involving division of
fraction by a whole number and vice versa.
1
Example 3:Four sisters share of a gallon of apple juice. What
3
fraction of a gallon of apple juice does each sister get?

Step 1: Write an equation.


1
n
÷4 =
3

Step 2: Write a related multiplication sentence. Then solve.

1 1
× = n
3 4
1
=n
12
1
Therefore, each girl will receive of a gallon of apple juice.
12

Exercises 1 – 7:

A- Divide.
3 1
1) 6 ÷ 2) ÷9
4 2

10 1
3) ÷2 4) ÷4
13 5

B- Solve.

5) Fouad prepared 3 sandwiches and he divides them into fourths. How


many ¼ - sized sandwiches does he have?

6) There are 32 members in the glee club. Deepak brought 12 cookies to


share with all of his co-members. How much cookies will each
member receive from Deepak? Solve using division and fractions.

7) Fadi has ¼ kilograms of strawberries. He distributes the fruits


equally into 4 bags. What fraction of kilograms of strawberries is in
each bag?

164
C
HAPTER 9
Chapter 9: Algebra: Patterns and
Graphing

165
Chapter 9 Lesson 1: Line Plots

GO MATH: Ch.9 L1

Vocabulary Keywords: Learning Outcomes:


-Draw a line plot
Line Plot -Use a line plot to solve problems.
Average

Mr. Ikram told his students that their average score in the midterm
exam is 88. What does Mr. Ikram mean by average score? The term
average means the central value of a set of numbers. Mr. Ikram got the
average of the scores by his students by dividing the sum of the scores
by the number of students. A line plot can help us find the average of a
given data set.

Definition: A line plot is a graphical representation that shows the


shape of the data by putting X marks above each data value on a number
line.

Example: Find the average length of insects using the data below.
Draw a line plot to assist you in solving.
1 1 1 3
inch, inch, inch, inch,
4 4 2 4
1 1 1 1
inch, inch, inch, inch,
4 4 4 2
1 3 1 3 1
inch, inch, inch, inch, inch
4 4 4 4 4

Step 1: Draw a line plot.

Start with writing the fractions on the number line. Then plot the given
fractions on the number line directly above their corresponding place on
the number line.

166
Step 2:Use the order of operations to find the average.

The number of X marks in the line plot indicates the number of fractions
in the data.

 1   1   3 
 8 × 4  +  2× 2  +  3× 4   ÷12 Perform the operations inside the parentheses.
     
8 9
 4 + 1 + 4  ÷12 Next, perform the opeartions in the brackets.
 
21
÷12 Divide.
4
7
Write the expression as the fraction.
16
7
Therefore, the average length of the insects is
16

Exercises 1 – 7:

A- Create a line plot then find the average of the data.

1)
1 3 3 1 1 3 3
Sticks 4 3 3 4 4 4 4 3 3
2 4 4 2 2 4 4

2)
1 2 2 3 1 2 2 2 3
Cups
5 5 5 5 5 5 5 5 5

167
B- Solve. Mr. Rashid, a meteorologist set up
rain gauges at various locations around a
certain town, and recorded the rainfall
amounts in the table below. Use the data in
the table to create a line plot using inches
and answer the following questions.

3) Which location receives the most rainfall?

4) Which location received the least rainfall?

5) Which rainfall measurement was the most


frequent?

6) What is the total rainfall in inches?

7) What is the average amount of rainfall?

168
Chapter 9 Lesson 2: Graph Data

GO MATH: Ch.9 L2,3

Vocabulary Keywords: Learning Outcomes:

x - axis ordered pair -Draw the graph of points on


y – axis origin a coordinate plane
x – coordinate -Use coordinate grid to
y – coordinate display data.

Locations on a map are recognized using coordinates represented by the


latitude and longitude. In algebra, we can use a coordinate grid in order
to locate points and be able to solve problems involving it.

Definition 1: The horizontal number line on the coordinate plane is


called the x – axis and the vertical line is called the y - axis.

Points on the coordinate plane can be represented by an ordered pair.

Definition 2: The first number in an ordered pair is called the


x – coordinate and the second number is called the y – coordinate.

Definition 3: The point of intersection of the x and y – axis is called the


origin located at (0, 0).

Example 1:Plot the point (5,6) and label it J.

A point on a coordinate plane can be named with


an ordered pair, a letter or both.

Starting from the origin, move 5units to the right


and then 6 units up.

Plot and label the point with point J.

169
Example 2: Find the distance between two points.

You can determine the distance between two points


when the points are along the same horizontal of
vertical line.

• Draw a line segment to connect point A and


point B.

• Count vertical units between the two points.

Since there are 6 vertical units, then the distance


between A and B is 6 units.

We can also use the coordinate plane in order to display the data
collected in an experiment.

Rule: If the data collected was measured in two ways, then write the
data as an ordered pair and graph then on the coordinate grid.

Example 3:Graph the data on a coordinate plane.

Height of Plants
Day 1 2 3 4
Height (cm) 5 7 8 9

Step 1: Choose a title for the data


Step 2: Use the data categories for the x – axis
and y – axis. Let the number of days be
represented by the x- axis and the height for the y
– axis.
Step 3: Write the ordered pairs.
(1, 5), (2, 7), 3, 8), (4, 9)
Step 4: Graph the points obtained in Step 3

170
Exercises 1 – 12:

A- Plot each point on the coordinate grid.

1) T( 3, 3 ) 2) S( 1, 8 )

3) H( 2, 8 ) 4) E( 6, 2 )

5) R( 5, 4 ) 6) L( 7, 6 )

7) M( 3, 1 ) 8) V( 9, 5 )

9) P( 7, 1 ) 10) A( 4, 7 )

B- Graph the data on a coordinate plane.

11)

Rayan’s Weight
Age 3 4 5 6
Weight (pounds) 31 35 40 46

12)
Rayan’s Height
Age 3 4 5 6
Height (inches) 33 37 40 41

171
Chapter 9 Lesson 3: Line Graphs

GO MATH: Ch.9 L4

Vocabulary Keywords: Learning Outcomes:

Line Graph -Draw line graph to display data.


Scale -Interpret data using a line graph.
Intervals

The temperature is changing over time. For example, the temperature in


the morning at 6:00 AM is 28o C and becomes hotter at 12 noon at 38oC.
To visualize the changes of temperature in a day, the best way is to use a
line graph.

Definition: A graph that uses line segments to show how data changes
over time is called a line graph. The set of numbers placed at fixed
distances that name the graph are the graph’s scale. The equal
differences between the values on the scale are called intervals.

Rule: To display data using a line graph, organize the data in ordered
pairs. Select a title, scale and intervals for the line graph. Plot the points
represented by ordered pairs then connect the consecutive points using
line segments.

Example: The data below shows the number of cars sold by Mr.
Ahmad from January to June 2015. Draw a line graph.

Mr. Ahmad’s Sales


Month Jan Feb Mar Apr May Jun
No. of Cars 20 35 25 40 45 30

A. On which month was the total number of cars sold the


greatest?

B. On which month was the total number of cars sold the least?

172
Step 1: Write the ordered pairs.
(Jan, 20), (Feb, 35), (Mar, 25), (Apr, 40), (May, 45), (Jun, 30)
Step 2:Choose the appropriate scale and interval
Step 3: Label the horizontal axis with Months and the vertical axis
with Number of Cars Sold. Write a title and graph the ordered pairs.
Join the consecutive points with line segments.

Mr. Ahmad’s Sales

Months

Exercises 1 – 2:

Make a line graph for the following data.

1) 2)

173
Chapter 9 Lesson 4: Graph and Analyze
Relationships
GO MATH: Ch.9 L5,6,7

Vocabulary Keywords: Learning Outcomes:


-Produce numerical patterns using a
Graph rule.
Patterns -Solve problems by finding the rule
Rules - Draw the graph of the relationship
Analyze between two numerical patterns on
Coordinate grid a coordinate plane

Drummers follow a distinct pattern in playing music through beats. If


you play drums and there is no rhythm, it is already considered as a
noise. Similarly, numbers also follow a certain pattern and sequence.

How are we going to identify if there is any relationship between two


numerical patterns?

Rule 1: Find a rule to write the first few terms in every sequence. Using
two sequences write number pairs and then determine a rule that
relates one sequence to the other.

Rule 2: Find a pattern using simple numbers and use a pattern that you
found to predict results with greater numbers to solve a problem.

Example 1: Write a rule that describes how one sequence is related


to the other sequence. Use the rule to determine the unknown
term.

Number of
1 2 3 4 8 10
Groups

Number of
12 24 36 48 96 ?
Players

Number of Balls 3 6 9 12 ? 27

174
Compare the number of players with the number of balls.

Notice that when you multiply the number of balls by 4, we get the
number of players.

For instance, if the number of balls is 3, the number of players is 12.


Therefore, one rule is:

“Multiply the number of balls by 4 to find the number of players.”

The other rule is:

“Divide the number of players by 4 to find the number of balls”

Now we can find the missing value in the data. Following the rules that
we generated, the completed table is shown below.

Number of
1 2 3 4 8 10
Groups

Number of
12 24 36 48 96 108
Players

Number of Balls 3 6 9 12 24 27

Let us now write and graph data on a coordinate grid using numerical
patterns.

Rule 3: To write and graph ordered pairs on a coordinate grid using


patterns, use the given rule in the problem to find the first few terms in
every pattern. Write the ordered pairs that represent the relationship in
the pattern. Graph and label the ordered pairs. If a line can be drawn
from the origin passing though all points on the grid, then the pattern
between two relationships can be obtained by multiplying.
175
Example 2:Maryam is working at the snack stand inside the
basketball arena. Each frozen yogurt costs 3AED. If one player
bought 8 cups of frozen yogurts, how much will he pay in all?

Step 1: Create a table of values for the number of yogurts and generate
the prices for each number of yogurts.

Number of
1 2 3 8
Yogurts

Price 3 6 9 ?

Step 2: Write a rule that describes the pattern.

From the table, when you multiply the number of yogurt by 3, you
will get its corresponding price. That is 1 x 3 = 3, 2 x 3 = 6, and 3 x 3 = 9.
Therefore the rule is” “Multiply the number of yogurt by 3”

Step 3: Write the ordered pairs.

(1, 3), (2, 6), (3, 9)

Step 4: Graph the relationship on a coordinate grid.

Step 5: Solve the problem.

Using the rule, multiply 8 cups of yogurt by 3. Therefore, the price


of 8 cups of yogurt is 24AED.

176
Exercises 1 – 3:

1) A rectangle has a pattern of 3 equal –sized squares each of which


is divided into 2 equal sized triangles. If Abdulla uses 36 of these
rectangles, how many triangles are there? Write a rule, and graph
the relationship.

2) Tala is buying an item worth 135AED. She will pay 15AED every
week until the item is paid. How much will she have left to pay
after 6 weeks? Write a rule, and graph the relationship.

3) Yousuf needs to purchase a wire whose length of 4 ½ feet long.


How many inches of wire will be needed? (Note that 1 foot = 12
inches)Write a rule, and graph the relationship.

177
178
10
C
HAPTER
Chapter 10: Convert Units of
Measure

179
Chapter 10 Lesson 1: Customary Conversions
in Multi-step Problems
GO MATH: Ch.10 L1,2,3,4

Vocabulary Keywords: Learning Outcomes:


Length foot -Compare, contrast and convert
Inch mile customary units of length
Capacity cup -Compare, contrast and convert
Fluid ounce customary units of capacity
Gallon pint -Compare, contrast and convert
Weight ton customary units of weight
Pound quart -Solve problems about conversion

Measurements are very important in our daily lives. We use


measurements to take the correct amount of medicine when we get sick,
to cook properly with the correct amount of ingredients, to buy clothes
that suits our size, to play sports well, to make estimations on the time
needed to prepare to go to school, to find out if your luggage meets the
requirement of the airline, for proper use of capacity, transportation and
for building structures. In this lesson, we will focus on different
customary measurements of length, capacity and weight.

Definition 1:Customary units are system of measurements that are


commonly used in the United States. These units are also known as the
English Units.

Definition 2:Length is the measurement or extent of something from


end to end

Customary Units of Length

1 foot ( ft ) = 12 inches ( in.)

1 yard ( yd ) = 3 ft

1 mile ( mi ) = 5,280 ft

1 mile = 1,760 yd

180
Definition 3:Capacity is the amount that something can hold.

Customary Units of Length

1 cup ( c ) = 8 fluid ounces ( fl oz )

1 pint ( pt ) = 2 cups

1 quart ( qt ) = 2 pints

1 gallon ( gal ) = 4 quarts

Definition 4:Weight is a measure of the heaviness of an object.

Customary Units of Weight

1 pound ( lb ) = 16 ounces ( oz )

1 ton ( T ) = 2,000 lb

Rule 1: To compare and convert customary units, use division on order


to change the smaller units to larger units or use multiplication to
change larger units to smaller units. Use the symbols <, > or = to
compare their measures.

Rule 2:Each relationship in the conversion tables can be written as a


unit ratio. You can multiply by a unit ratio to convert units.

Some examples of unit ratio are as follows:

3 ft 2000lb 4qt
1 yd 1T 1gal

181
Example 1:Convert larger units to smaller units.

A. Convert 24 feet to inches.


12in.
Since 1 ft = 12 in., then we can use the unit ratio
1 ft.

12in 12in.
=
24 ft 24 ft • Multiply by
1 ft 1 ft.

=
24 ft 24 ft •
12in Divide out common units.
1 ft

24 ft =24 • 12in =288in. Multiply.

Therefore, 24 feet = 288 inches.

B. Omar mixes 1/3cup of fertilizer with soil on the clay pots


before planting the seed. How many ounces of fertilizer does
he use per pot?
8 fl oz.
Since 1 cup = 8 fluid ounces, then we can use the unit ratio
1c.

1 1 8 fl oz 8 fl oz.
=c c• Multiply by then divide out
3 3 1c 1c.
common units.
1 1
c=
8
• 8 fl oz = fl oz Multiply
3 3 3

Therefore, 8/3 fluid ounces of fertilizer are used per clay pot.

182
Example 2:Convert smaller units to larger units.

A. Convert 20 quarts into gallons.


4qt
Since 1 gallon = 4 quarts, then the unit ratio is and its
1gal
1gal
reciprocal is
4qt

1gal 1gal
=
20 qt 20qt • Multiply by
4qt 4qt
=
20 qt 20qt •
1gal Divide out common units
4qt
20qt =
1
20 • gal = 5 gal Divide by 4
4

Therefore, 20 quarts = 5 gallons

B. A 64-ounce beef belly can be cut into 10 smaller beef chops of


equal weight. How many pounds does each beef chop weigh?
16oz
Since 1 pound = 16 ounces, then the unit ratio is and its
1lb
1lb
reciprocal is
16oz

Each beef chops in ounces is 64 oz ÷10 = 64/10 oz. or 32/5 oz.

32 32 1lb 1lb
= oz oz • Multiply by
5 5 16oz 16oz

32
= oz
32
oz •
1lb Divide out common units
5 5 16oz

32
oz=
32lb
÷ 16=
2
lb Divide
5 5 5

183
Example 3:There are 8 jars of chocolate ice cream delivered to
Salman’s house for his party. Each jar contains 3 gallons of ice
cream. If each party guest is served with 1 cup of chocolate ice
cream, how many guests can be served?

Step 1: Write the given information.

8 jars of ice cream; each jar has 3 gallons of ice cream; each guest is
served with 1 cup of ice cream

Step 2: Determine the total amount of ice cream

8 x 3 = 24 gallons of ice cream

Step 3: Convert from gallons to cups.

Plan: Convert gal qt, qtpt, ptc

24 gal •
4qt 2 pt 2c
• •
Multiply with unit ratios until cups.
1gal 1qt 1 pt
24 gal •
4qt 2 pt 2c
• •
Divide out common units
1gal 1qt 1 pt
24 • 4 • 2 • 2 = 384 Multiply

Therefore, there are 384 guests can be served.

Exercises 1 – 17:

A- Complete the following.

1) 14ft = ____yd 2) 4 gal = ____qt

3) 2 mi = ____ft 4) 13 c = ____pt

5) 72 oz = _____lb 6) 3 ½ pt = ____c

7) 3 ¾ qt = ____pt 8) 2lb = ____oz

9) 3c = ____ fl oz 10) 7000 lb = ____T

184
B- Fill each blank with <, >, or = to make a true statement.

11) 16 in ____ 1 ½ ft 12) 9 gal ____32qt

13) 2.7 T ____ 86,400 oz 14) 36 in ____12 ft

C- Solve.

15) One of the largest pumpkins weighed about ½ ton. How many
pounds did the pumpkin weigh?

16) A 50-foot speed boat is for sale by Mr. Nasser. About how long is
this boat to the nearest yard?

17) A certain Web site recommends that you soak fabrics in a


mixture of three-fourths cup vinegar, 2 quarts water, and some
salt in order to whiten fabrics. Does a mixture that contains 1.5
ounces vinegar and 16 ounces water have the same vinegar-to-
water ratio as the recommended mixture? Justify your answer.

185
Chapter 10 Lesson 2: Problem Solving:
Customary and Metric
Conversions
GO MATH: Ch.10 L5,6

Vocabulary Keywords: Learning Outcomes:


- Compare and convert metric
Metric system kilo- units
Hecto- deka- -Solve problems involving
Meter deci- customary and metric units
Centi- milli-

In the previous lesson, we have learned how to convert customary units


of length, capacity and weight. In this lesson, we will deal with metric
system as well as the combination of metric and customary units of
measurement.

Definition: A decimal system of measures is called the metric system.

The following table shows many of the prefixes that you may
encounter:

186
These prefixes can be attached to different basic metric units. Watch
how this happens in the following list of some of the most common
metric units:

Note: The basic unit of length is the meter. The basic unit of capacity is
the liter. The basic unit of weight is kilogram.

Rule: The basis of the metric system is on place value. Every unit is
related to the next largest or next smallest unit by a power of 10.

Example 1:Convert metric units.

A. Convert 6.5 liters to milliliters.

Since 1 L = 1,000 mL, we have,

1 L = 1,000 mL

6.5 x 1L = 6.5 x 1,000mL

6.5 L = 6,500 mL

187
B. Convert 300 millimeters to meters.

1 mm = 0.001 m

300 x 1mm = 300 x 0.001 m

300 mm = 0.3 m

Example 2:The heaviest weight of a bear is 521.64 kilograms. What


is this weight in grams?

1 kg = 1,000g

521.64 x 1 kg = 521.64 x 1,000g

521.64 kg = 521,640 g

Therefore, the heaviest weight of the bear is 521,640 grams.

The following tables show the relationships between some English or


Customary and metric units. When converting between the two
systems, you will need to use these relationships to create the
fraction(s) that will make your initial unit cancel out to produce the
new unit.

188
Example 3:Convert between measurement systems.

Convert 15.24 inches to centimeters. Round off your answer to the


nearest hundredths.

1 in = 2.54 cm

15.24 x 1 in = 15.24 x 2.54

15.24 in = 38.71cm

Therefore, 15.24 inches is approximately 38.71centimeters

Example 4:The faucet of Huda’s toilet was dripping at a rate of 24


centiliters per day. How many milliliters of water will have dripped
from his faucet in 48 hours?

Since there are 10 milliliters in 1 centiliter and 48 hours means 2 days,


we have,

cL 1 2 3 48

mL 10 20 30 480

Therefore, 480 milliliters of water have dripped from Huda’s faucet in


48 hours.

Exercises 1 – 14:

A- Fill in the blank with the correct value to make the statement
true. Round off your answer up to 2 decimal places.

1) 820 cm =_____m 2) 982 mm = ____m

3) 4.3m = _____cm 4) 0.09 g = ____mg

5) 998 g =____kg 6) 23.56 kL = ____L

7) 12.2 L = ____mL 8) 22.23 in = ____cm

9) 154.23 lb = ____kg 10) 3.87 m = ____ ft

189
B-Solve.

11) At 828 meters tall, Burj Khalifa is the highest building in the
world. How many kilometers tall is the building?

12) A20-ounce jar contains 510 grams of grape jelly. How many
kilograms of grape jelly does the jar contain?

13) Aisha needs a 2.5-meter pole for a birdfeeder that she is


building. How many centimeters will she need to cut off a 3-meter
pole in order to use it for the birdfeeder?

14) Mohammad has a roll of paper that is 800 cm long. He wants to


divide the paper into 1-m strips. How many 1 – meter strips can
she cut?

190
Chapter 10 Lesson 3: Elapsed Time

GO MATH: Ch.10 L7

Vocabulary Keywords: Learning Outcomes:


- Solve elapsed time problems
Elapsed Time Hour involving conversion of time.
Seconds Minutes -Change time units.

Suppose you started studying for math exam from 5:00 PM to 9:30 PM,
how long did you study? To solve this problem, we simply have to find
the elapsed time.

Definition: The amount of time that passes from the start of an event to
the end is called an elapsed time.

Units of Time

60 seconds ( s ) = 1 min ( min )

60 minutes = 1 hour ( hr )

24 hours = 1 day ( d )

7 days = 1 week ( wk )

52 weeks = 1 year ( yr )

12 months ( mo ) = 1 year

365 days = 1 year

Rule: To solve elapsed time problems by converting units of time, use


multiplication or division and the conversion table to convert units then
add or subtract units of time.
191
Example: Starting at 3:20 PM, Latifa practiced her guitar for 100
mins. At what time did Latifa stop practicing?

Step 1: Convert minutes to hours

60 mins = 1hr

100 mins = 1hr 40 mins

Step 2: Count forward by hours until you reach 1 hr

3:20 PM  4:20 PM = 1 hr

Step 3: Count forward by minutes until you reach 40 mins

4:20 PM  4:30 PM = 1 hr 10 mins

4:30 PM  4:40 PM = 1 hr 20 mins

4:40 PM  4:50 PM = 1 hr 30 mins

4:50 PM  5:00 PM = 1 hr 40 mins

Therefore, Latifa stops practicing at 5:00 PM

Exercises 1 – 5:

Complete the table.

Start Time End Time Elapsed Time

1) 7:00 AM 13 hours

2) 8:30 AM 19 hours 30 mins

3) 2:15 PM 8:00 PM

4) 9:30 PM 15 hours 30 mins

5) 7:51 PM 11 hours 12 mins

192
11
C
HAPTER
Chapter 11: Geometry and Volume

193
Chapter 11 Lesson 1: Polygons

GO MATH: Ch.11 L1

Vocabulary Keywords: Learning Outcomes:


Polygon - Identify polygons.
Regular Polygon -Classify polygons.
Congruent

The Pentagon is the main headquarters of the US


Department of Defense. The shape of this cross-
section of this structure is a two dimensional
figure with 5 sides. In general, how are we going
to name and classify shapes if the number of sides
is 4, 6, 7 or 8 or n?

Definition 1:A polygon is a closed plane figure with straight line sides
which do not cross over.

Polygon
Triangle Quadrilateral Pentagon Hexagon
No. of Sides 3 4 5 6
No. of
3 4 5 6
Angles
No. of
3 4 5 6
Vertices

Polygon
Heptagon Octagon Nonagon Decagon
No. of Sides 7 8 9 10
No. of
7 8 9 10
Angles
No. of
7 8 9 10
Vertices
194
Definition 2:A regular polygon has all sides of equal length and all
angles are congruent. Otherwise, it is called irregular polygon.

Regular Polygon Irregular Polygon

Definition 3:A convex polygon is a polygon with no interior reflex


angles.

Exercises 1 – 5:

A- Name each polygon and identify which one is regular and


which one is not.

1) 2)

3) 4)

B- 5)Explain why these figures are not regular polygons:

195
Chapter 11 Lesson 2: Triangles

GO MATH: Ch.11 L2

Vocabulary Keywords: Learning Outcomes:


Equilateral Triangles - Classify triangles using their
Scalene Triangle properties.
Isosceles Triangle - Draw triangles using their
properties.

The Bermuda Triangle is called as such


because it is of the shape of a triangle
whose vertices represent the Bermuda
Island, Florida and Puerto Rico.

Definition 1:A triangle is a three sided polygon. It has three vertices,


three sides and three angles.

Triangles can be classified based on special characteristics of their sides


or their angles

Based on the sides, there are three different types of triangles

Scalene Triangle: no equal sides

Isosceles Triangle: two equal sides

196
Equilateral Triangle: all equal sides

Based on the angles, there are three different types of triangles

Acute Triangle: a triangle with every angle that measures less than 90°

Right Triangle: a triangle with an angle that measures exactly 90°

Obtuse Triangle: a triangle with an angle that measures greater than


90°

197
Example: Check the sides measurement and classify the triangles.
Write isosceles, scalene, or equilateral.

1) Maher draws a triangle of three different sides: 4 cm, 6 cm and 5


cm. The triangle is scalene.

2) Saleh draws a triangle of three similar sides: 7 cm, 7cm, 7 cm. The
triangle is equilateral.

3) Majed draws a triangle of two similar sides: 9 cm, 9 cm, 3 cm. The
triangle is isosceles.

Exercises 1-9:Classify triangles as isosceles, scalene, and


equilateral if the sides are given. Classify triangles as right, obtuse,
acute if the angles are mentioned.

1) 2)

3) 4)

5) 6)

7) 8)

9)

198
Chapter 11 Lesson 3: Quadrilaterals

GO MATH: Ch.11 L3,4

Vocabulary Keywords: Learning Outcomes:


Trapezoid -Classify polygons according to the
Parallelogram properties of their sides.
Rectangle -Classify polygons according to the
Square properties of their angles.
Rhombus

Polygons that have 4 sides and 4 angles are quadrilaterals.


Quadrilaterals are in our everyday life and are often relied up in
construction and design.

Quadrilaterals can be classified by looking at the number of parallel


sides, the lengths of their sides, and the measures of their angles.

Trapezoid Parallelogram Rectangle Rhombus Square


Number of
parallel
sides 1 pair 2 pairs 2 pairs 2 pairs 2
pairs

Number of
congruent
sides 0 pair 2 pairs 2 pairs all 4 all 4
sides sides

Number of
congruent
angles 0 pair 2 pairs all 4 2 pairs all 4
angles angles

199
To classify the quadrilateral at the right,
identify the following characteristics.
6m 140° 6m

40° 40
Number of parallel sides: 2 pairs
6m 140° 6m
Number of congruent sides: all 4 sides

Number of congruent angles: 2 pairs

Example: If Aisha needs to draw a square, what will be the number


of the parallel sides, congruent sides and congruent angles?

Number of parallel sides: 2 pairs


Number of congruent sides: 4 sides
Number of congruent angles: 4 angles

Note: To estimate if the sides of the figure are congruent, fold the figure
as needed to decide if the sides are of the same length.

Exercises 1 – 8:

State the most specific name for each figure.


1) 2)

3) 4)

200
C- Divide each polygon to the given number of congruent
triangles.

5) 4 congruent triangles 6) 8 congruent triangles

7) 6 congruent triangles 8) 8 congruent triangles

201
Chapter 11 Lesson 4: Solid Figures and
Volume
GO MATH: Ch.11 L5,6,7,8,9,10

Vocabulary Keywords: Learning Outcomes:


Polyhedron - Identify, classify and describe three
Prism dimensional figures
Base -Use a unit cube to build a solid figure
Lateral face -Count unit cubes to find volume
Pyramid -Solve problems about volume
Volume

Consider the following objects.

These objects are three-dimensional figures. It is called as such because


of the fact that they have three dimensions – length, width and height.
These are also called solid figures.

Definition 1:Three-dimensional figures whose faces are polygons are


called polyhedrons.

Definition 2:A polyhedron having two congruent


polygons as base is called prism. The lateral faces are
the rectangles that connect the two bases of the
prisms.

A prism is named according to the shape of its base. For instance, the
prism above is called a triangular prism.

Types of Prisms

• Decagonal Prism • Pentagonal Prism


• Octagonal Prism • Rectangular Prism
• Hexagonal Prism • Triangular Prism

202
Definition3:A prism whose bases and lateral faces are congruent is
called a cube.

Definition4:A polyhedron having one base is called a pyramid. The


lateral faces of a pyramid are triangles that intersect at a common
vertex. Pyramids are also named according to its base.

Types of Pyramids
• Pentagonal Pyramid
• Rectangular Pyramid
• Square Pyramid
• Triangular Pyramid

Definition 5:Solid figures having curved surfaces are called


non-polyhedrons. The figures that are considered as non-polyhedrons
are as follows:

Definition 6:A solid figure that has one circular base and
one curved surface is called a cone.

Definition 7:A solid figure that has a pair of congruent


circular bases and one curved surface is called a cylinder.

Definition 8:A solid figure having no bases and curved


surface is called a sphere. .

One of the attributes of a solid figure is the volume.

Definition 9:The amount of space occupied by an object


or solid figures is called the volume. The volume is
measured according to the number of unit cubes. A
cube whose length, width, and height are equal to 1
unit is called a unit cube. Each unit cube has a volume
of 1 cubic unit.
203
A cube has 6 congruent squares faces. It has 12 edges. The lengths of all
its edges are also congruent.

Rule 1: Count the number of unit cubes to find the volume of rectangular
prisms.

Rule 2:To find the volume of solid figures, measure the solid in three
directions. In the case of a rectangular prism, measure its length, width,
and height. The units of measurement used for volume are: cu cm, cu in,
or cu ft.

Example 1:Find the volume of the prism below by counting the unit
cubes.

This is a rectangular prism. We can find its volume by counting the unit
cubes.

We can visualize this prism as shown below.

Since there are 24 unit cubes in the solid, then its volume is equal to 24
cubic units.

204
Example 2:Jamal packs boxes into a large box. Estimate the volume
of the large box.

In the figure, each small box is a unit cube.


Considering the number of unit cubes that can fit
inside the large box, we can say that the volume of
the large box is 60 cubic units.

Rule 3:Prisms with the same dimensions have equal volumes.

Rule 4: The formula to find the volume of a rectangular prism is given


below.
Volume = Base area × height
V = B×h
B = area of the base
h = height
Volume = length × width × height
V = l×w ×h

Rule 5:Another formula for the volume of prisms is given below..

Volume = Base area × height


V = B×h
B = area of the base
h = height

Example 3:Find the volume.

Method 1: Using V = l × w × h

Step 1: Identify the dimensions of the prism.


l = 5 inches
w = 4 inches
h = 3 inches

205
Step 2: Multiply the length by the width.
l x w = 5 x 4 = 20

Step 3: Multiply the product of the length and width by the height.
20x 3 = 60

Therefore, the volume of the prism is 60 cubic inches.

Method 2: Using V = B × h

Step 1:Find the area of the base.


Since the base is a rectangle, then the area = l x w = 5 x 4 = 20

Step 2:Substitute B and h with the height of the solid figure.


V = Bh
V = 20 x 3

Step 3: Multiply.
V = 60 cubic inches.

Exercises 1 – 10:

A- Classify the following solid figures. If the figure is a prism or


pyramid, name them according to their base/s.

1) 2)

3) 4)

206
B- Find the volume of the prisms using the unit cubes.

5) 6)

C- Find the volume of the following rectangular prisms.

7) 8)

9) 10)

207
Chapter 11 Lesson 5: Comparing Volumes and
Composite Figures
GO MATH: Ch.11 L11,12

Vocabulary Keywords: Learning Outcomes:


Volume - Compare different
Composite Figures rectangular prisms with equal
volumes.
-Calculate the volume of
bi d t l i

Last time, we have learned how to find the volume of rectangular


prisms. For instance if there are 20 small boxes are inside a large box
with no spaces, then the volume of the large box is equal to the total
number of small boxes (the unit cubes) which is 20. In this lesson, we
are going to apply the volume formulas in some applications. One is in
comparing volumes of rectangular prisms, and another application for
the combination of rectangular prisms.

Rule 1: To compare rectangular prisms with the same volumes, follow


the following steps:

1) Read the problem carefully and find out what is required and
what information are needed to solve the problem.

2) Create a table using the given information

3) Use the table in order to find the answer to the problem.

Example 1:A factory in Al Quoz is making three different sizes of


rectangular boxes. The length of each box is two times the width
and height. If the heights of the three boxes are 2, 3 and 4 feet, what
is the volume of each box?

208
Step 1: We need to determine the volume of each box. The given
information are the heights of the three boxes and that the length is
twice the width and height.

Step 2: List down the corresponding values of h. The widths are equal to
the heights because they are both equal to ½ of the corresponding
lengths. Find the length by multiplying each width or height by 2. Lastly,
find the volume of each using the formula for volume.

Step 3: Solve the problem.

length width height Volume

4 2 2 16

6 3 3 54

8 4 4 128

Therefore, the volumes are 16 cubic feet, 54 cubic feet and 128 cubic
feet.

Now, let us use the volume formulas to find the volume of composite
figures.

Example 2: Use addition to find the volume of the composite figure


below.

209
Step 1: Break apart the figure into two rectangular prisms.

Step 2: Find the length, width, and height of each prism.

Prism 1: l = 7 cm; w = 2 cm; h = 2 cm

Prism 2: l = 2 cm; w = 2 cm; h = 5 cm

Step 3: Find the volume of each prism.

Volume (Prism 1) = l x w x h = 7 x 2 x 2 = 28 cubic centimeters

Volume (Prism 2) = l x w x h = 2 x 2 x 5 = 20 cubic centimeters

Step 4: Add the volumes of Prism 1 and Prism 2.

28 + 20 = 48 cubic centimeters.

Therefore, the volume of the composite figure is 48 cubic centimeters.

Example 3: Use subtraction to find the volume of the composite


figure

210
You can subtract the volumes of prisms formed in empty spaces from
the greatest possible volume to determine the volume of a composite
figure.

Step 1: Find the greatest possible volume.

Volume of greatest possible volume = 80mm x 10mm x 60mm

= 48,000 cubic millimeters

Step 2: Find the volume of the prism in the empty space.

Volume of the Empty Space = 60 mm x 10mm x 40 mm

= 24,000 cubic millimeters

Step 3: Subtract the two volumes.

Volume of Composite Figures = 48,000 – 24,000 = 24,000 mm3

211
Exercises 1 – 5:

A- Solve.

1) Ali is moving to a new flat and he needs a box for his school
supplies. The first box in the store has the dimensions 12 in by 4
in by 5 in. and the other box has 10 in by 5 in by 6 in. He wants to
choose the box with a greater volume. Which box should he
choose?

2) A company is producing water beds with four different sizes


which are of the shape of rectangular prisms. The length of each
bed is three times the width and three times the height. The
heights of the beds are 2, 3 4, and 5 feet. If the beds are filled all
the way to the top, find the volume of each bed.

B- Find the volume of the composite figures.

3)

4)

5)
212
213

You might also like